Chapter 24: Structure and Function of the Kidney Chapter 25: Disorders of Renal Function- Chapter 26: Acute Kidney Injury and Chronic Kidney Disease.

Lakukan tugas rumah & ujian kamu dengan baik sekarang menggunakan Quizwiz!

A 35-year-old client is diagnosed with acute kidney injury (AKI) and is started on hemodialysis. The client is concerned with the diagnosis and wants to know what to expect in the progression of this disorder. Which statement best addresses the client's concern? a) "You will need to have a renal transplant to live a productive, healthy life." b) "Once your condition improves you can be placed on peritoneal dialysis for the rest of your life." c) "The occurrence of acute kidney injury will always eventually result in chronic renal failure." d) "Acute kidney injury is abrupt in onset and often reversible if recognized early and treated appropriately."

"Acute kidney injury is abrupt in onset and often reversible if recognized early and treated appropriately." AKI is the result of a rapid decline in kidney function that increases blood levels of nitrogenous wastes. Once the underlying cause is treated, AKI is potentially reversible if the precipitating factors can be corrected or removed before permanent damage occurs.

The nursing instructor who is teaching about disorders of the lower urinary tract realizes a need for further instruction when one of the students makes which of the following statements? a) "Alterations in bladder function can include urinary obstruction with retention or stasis of urine." b) "Alterations in bladder function occurs frequently in the elderly." c) "Alterations in bladder function can only occur when there is incontinence." d) "Alterations in bladder function can include urinary incontinence with involuntary loss of urine."

"Alterations in bladder function can only occur when there is incontinence." Correct Explanation: Alterations in bladder function include urinary obstruction with retention or stasis of urine and urinary incontinence with involuntary loss of urine. Alterations in bladder function does occur more frequently with aging.

The nurse is scheduled to teach a client experiencing urinary incontinence about Kegel exercises. Which of the following descriptors should the nurse include in this education? a) "After you have emptied your bladder, continue sitting on the commode and try to forcefully expel more urine." b) "Contract and relax the pelvic floor muscles at least 10 times every hour while awake." c) "Drink at least two glasses of water and then try to hold it for at least 3 hours before going to the bathroom." d) "Try to start and stop urination while sitting in a bathtub full of warm soapy water."

"Contract and relax the pelvic floor muscles at least 10 times every hour while awake." Correct Explanation: Exercises for the pelvic muscles or Kegel exercises involve repetitive contraction and relaxation of the pelvic floor muscles and are an essential component of client-dependent behavioral interventions. None of the other distractors are examples of Kegel exercises.

Which substance, released by the atria, causes vasodilation of the afferent and efferent arterioles, which results in an increase in renal blood flow and glomerular filtration rate (GFR)? a) Antidiuretic hormone (ADH) b) Angiotensin I c) Atrial natriuretic peptide (ANP) d) Aldosterone

Atrial natriuretic peptide (ANP) Explanation: Atrial natriuretic peptide is a hormone that is synthesized in the muscle cells of the atria of the heart and released when the atria are stretched. The actions of ANP include vasodilation of the afferent and efferent arterioles, which results in an increase in renal blood flow and glomerular filtration rate. Angiotensin I, which has few vasoconstrictor properties, leaves the kidneys and enters the circulation. ADH and aldosterone would decrease blood flow and GFR

The nurse suspects that a newborn infant who presents with bilateral flank masses, impaired lung development, and oliguria may be suffering from which of the following disorders? a) Autosomal recessive polycystic kidney disease (ARPD) b) Simple renal cysts c) Horseshoe kidney abnormality d) Autosomal dominant polycystic kidney disease

Autosomal recessive polycystic kidney disease (ARPD) The typical infant with ARPD presents with bilateral flank masses, accompanied by severe renal failure, signs of impaired lung development, and variable degrees of liver fibrosis and portal hypertension.

A patient has just been diagnosed with acute glomerulonephritis. Which question should the nurse ask this client in attempting to establish a cause? a) "Have you had any type of infection within the last 2 weeks?" b) "Do you have a history of heart failure?" c) "Have you ever been diagnosed with diabetes?" d) "Have you recently had kidney stones?"

"Have you had any type of infection within the last 2 weeks?" Acute post-infectious Glomerulonephritis usually occurs after infection with certain strains of group A β-hemolytic streptococci and is caused by deposition of immune complexes of antibody and bacterial antigens. Other organism can also cause this infection.

A 42 year-old male has been diagnosed with renal failure secondary to diabetes mellitus and is scheduled to begin dialysis soon. Which of the following statements by the client reflects an accurate understanding of the process of hemodialysis? a) "I won't be able to go about my normal routine during treatment." b) "It's stressful knowing that committing to dialysis means I can't qualify for a kidney transplant." c) "Changing my schedule to accommodate 3 or 4 hours of hemodialysis each day will be difficult." d) "I know I'll have to go to a hospital or dialysis center for treatment."

"I won't be able to go about my normal routine during treatment." Hemodialysis requires the client to remain connected to dialysis machinery, whereas peritoneal dialysis allows for activity during treatment. -Dialysis does not disqualify an individual from receiving a transplant. -Dialysis does not require attendance at a dialysis center, patients can be taught to perform the dialysis in their home with a family member in attendance. -Hemodialysis is normally conducted 3 times weekly, not once per day.

A nurse educator is performing client education with a 51 year-old man who has been recently diagnosed with chronic kidney disease. Which of the following statements by the client would the nurse most likely want to correct or clarify? a) "My heart rate might go up because of my kidney disease and my blood might be a lot thinner than it should be." b) "My kidney problems increase my chance of developing high blood pressure or diabetes." c) "I'll be prone to anemia, since I'm not producing as much of the hormone that causes my bones to produce red blood cells." d) "I'll have a risk of either bleeding too easily or possibly clotting too quickly, though dialysis can help minimize these effects."

"My kidney problems increase my chance of developing high blood pressure or diabetes." While high blood pressure can be causative of- or consequent to- renal failure, diabetes is not normally a result of existing CKD. Persons with renal failure are indeed prone to anemia, increased heart rate, decreased blood viscosity, and coagulopathies. The risk of bleeding and thrombotic disorders can be partially mitigated by dialysis

A client who has had recurrent UTIs asks the nurse about the old wise tale of drinking cranberry juice daily. The nurse can respond: a) "There is no research on this topic, so I don't think it will help you." b) "Studies on this are based on a person drinking at least 1 gallon of juice/day." c) "Research suggests cranberry juice will reduce bacterial adherence to the lining of the urinary tract." d) "Beer is probably more effective at killing bacteria than cranberry juice."

"Research suggests cranberry juice will reduce bacterial adherence to the lining of the urinary tract." Correct Explanation: Cranberry juice or blueberry juice has been suggested as a preventive measure for persons with frequent UTIs. Studies suggest that these juices reduce bacterial adherence to the epithelial lining of the urinary tract.

The client has just been diagnosed with bladder cancer and asks the nurse what causes it. Which of the following would be the nurse's best response to the client? a) "The cause is unknown." b) "It is usually inherited." c) "It is due to something outside the body." d) "You should not worry about its cause."

"The cause is unknown." The nurse should tell the patient that the cause of bladder cancer is unknown; however, evidence suggests that its origin is due to local influences such as carcinogens that are excreted in the urine and stored in the bladder. It is not inherited and it would not be therapeutic to tell the patient not to be concerned about its cause.

A bladder cancer client asks the nurse, "What did the doctor mean by intravesicular chemotherapy? Am I going to lose all my hair and have to do for treatments over months and months?" The best response would be: a) "This is when they use a CyberKnife to cut off any lesions and then inject chemotherapy into the remaining portion of the bladder." b) "They will take you to radiology and inject some chemotherapy through your abdomen into your bladder." c) "The doctor will place a scope up your urethra, into the bladder, and burn the lining of the bladder with a laser and then inject some tuberculosis bacillus into the lining." d) "This is when they put the chemotherapy directly into the bladder to kill any cancer cells."

"This is when they put the chemotherapy directly into the bladder to kill any cancer cells." Correct Explanation: Surgical treatment of superficial bladder cancer is often followed by intravesicular chemotherapy or immunotherapy, a procedure in which the therapeutic agent is directly instilled into the bladder. None of the other responses describe this procedure. The chemotherapy drug is not injected through the abdomen into the bladder. BCG is instilled into the bladder to elicit an inflammatory response that can kill the tumor. A CyberKnife is used with the brain, not the bladder

A patient is scheduled for a creatinine clearance test to measure the glomerular filtration rate (GFR). The patient asks the nurse what this test is used for. Which of the following is the nurse's best response? a) "This test measures urine-specific gravity." b) "This test provides a gauge of renal function." c) "This test measures the volume of urine output." d) "This test measures circulating antidiuretic hormone levels."

"This test provides a gauge of renal function." Explanation: The GFR provides a gauge of renal function. The GFR is the amount of filtrate that is formed each minute as blood moves through the glomeruli. The clearance rate for creatinine is the amount that is completely cleared by the kidneys in 1 minute.

A patient has a postvoid residual (PVR) volume of 250 mL. Which of the following information would the nurse tell the patient? a) "This test indicates you do not have adequate bladder control." b) "This value indicates you are emptying your bladder too completely." c) "This value indicates you are having difficulty emptying your bladder." d) "This is a normal value."

"This value indicates you are having difficulty emptying your bladder." Correct Explanation: The nurse should inform the client that values over 200 mL indicate that the patient is not able to adequately empty the bladder and that further evaluation is necessary. It is not a normal value, nor does it indicate that the patient does or does not have adequate bladder control. (

The clinical nurse educator on a nephrology unit of a large, urban hospital is orientating recent nursing graduates to the unit. Which of the following teaching points about acute tubular necrosis (ATN) should the educator include in the orientation session? a) "Trauma, burns and major surgery are common precursors to ATN." b) "The cardinal signs of ATN are oliguria and retention of potassium, creatinine and sulfates." c) "Ureteral and bladder outlet obstructions are often contributors to ATN." d) "Tubular epithelial cells are sensitive to ischemia and toxins, and damage is irreversible."

"Trauma, burns and major surgery are common precursors to ATN." ATN is often preceded by major surgery, burns or trauma. Many cases of ATN are nonoliguric, and obstructions that are postrenal in nature are not common causes of ATN. Damage to tubular epithelial cells is not necessarily irreversible.

A nurse is collecting a urine specimen prior to measuring the albumin level in a client's urine. A colleague questions the rationale for the test, stating, "I thought albumin was related to liver function, not kidney function." How can the nurse best respond to this statement? a) "A urine test for albumin allows us to estimate the client's GFR quite accurately." b) "Urine should normally be free of any proteins, and albumin is one of the more common proteins to be excreted in chronic renal failure." c) "Urine albumin levels are useful for diagnosing diabetic kidney disease." d) "A urine dipstick test will tell us exactly how much albumin is being spilled by the client's kidneys."

"Urine albumin levels are useful for diagnosing diabetic kidney disease." In clients with diabetes, albumin tests are useful adjunctive test of nephron injury and repair. -Urine is not normally completely free of proteins and a urine dipstick does not allow for the quantification of how much albumin is in a sample. -Albumin tests do not allow for an accurate indirect indication of GFR.

A nurse is collecting a urine specimen prior to measuring the albumin level in a client's urine. A colleague questions the rationale for the test, stating, "I thought albumin was related to liver function, not kidney function." How can the nurse best respond to this statement?

"Urine albumin levels are useful for diagnosing diabetic kidney disease." Explanation: In clients with diabetes, albumin tests are useful adjunctive test of nephron injury and repair. Urine is not normally completely free of proteins and a urine dipstick does not allow for the quantification of how much albumin is in a sample. Albumin tests do not allow for an accurate indirect indication of GFR.

A family physician is providing care for a 61 year-old obese male who has a history of diabetes and hypertension. Blood work has indicated that the man has a GFR of 51 mL/min with elevated serum creatinine levels. Which of the following statements will the physician most likely provide to the client in light of these results? a) "You're in kidney failure and I'll be starting dialysis treatment immediately." b) "Your chronic kidney disease has likely been caused by your diabetes and high blood pressure." c) "You likely have chronic kidney disease and there may be urine in your blood until it is controlled." d) "We will regularly monitor your kidney function, but most likely your kidneys will be able to compensate on their own and intervention is not required."

"Your chronic kidney disease has likely been caused by your diabetes and high blood pressure." Diabetes and hypertension are conditions that can cause chronic kidney disease (CKD). While the kidneys do have a remarkable ability to compensate for impaired function, this fact does not mean that treatment would not be undertaken. -Hematuria is not a common manifestation of CKD and the clients GFR of 51 mL/min does not indicate kidney failure or the need for dialysis.

A 34 year-old man has been taking up to 2400 mg of ibuprofen per day following a motor vehicle several months ago and consequent chronic pain. He has recently been diagnosed with chronic analgesic nephritis as a result of his high analgesic intake. The man is surprised at the diagnosis, stating, "I thought that taking too many drugs hurt your liver if anything, not your kidneys." What is the most appropriate response to the man's statement? a) "Your liver does perform most of the detoxifying in your body, but your kidneys can perform this role if the liver is unable to." b) "Your kidneys are vulnerable to damage because of how much blood flows through them and the fact that they break down many drugs." c) "High drug intake can cause your kidneys to be very vulnerable to infections, which is likely what happened in your case." d) "It is very rare for someone as young as yourself to have kidney damage like this; usually only older people are vulnerable to kidney damage from drugs."

"Your kidneys are vulnerable to damage because of how much blood flows through them and the fact that they break down many drugs." High flow and pressure combined with the metabolic transformative of drugs makes the kidneys vulnerable to drug toxicity. They do not play a backup role to the liver in the metabolism of drugs, and while structural and functional damage may occur with drug overuse, infection is a less likely consequence. The elderly are particularly vulnerable to drug-related nephritis and nephropathies, but it would be incorrect to conclude that younger people are thus immune.

Select the percentage of cardiac output that perfuses the kidneys. a) 10% to 15% b) 20% to 25% c) 15% to 20% d) 25% to 30%

20% to 25% Explanation: In the adult, the kidneys are perfused with 1000 to 1300 mL of blood per minute, or 20% to 25% of the cardiac output; 10% to 15% and 15% to 20% represent a lower percentage; 25% to 30% is an increased percentage

The nurse is planning care for a client with chronic kidney disease (CKD). The nurse determines that an allowable fluid intake would be: a) 500 to 800 mL/day b) 1050 to 1200 mL/day c) 400 to 600 mL/day d) 850 to 1000 mL/day

500 to 800 mL/day It is a common practice to allow 500 to 800 mL, which is equal to insensible water loss plus a quantity equal to the 24-hour urine output. -Fluid intake in excess of what the kidneys can excrete causes circulatory overload, edema, and water intoxication. -Inadequate intake, on the other hand, causes volume depletion and hypotension and can cause further decreases in the already compromised GFR.

The nurse knows that a patient with which glomerular filtration rate (GFR) would be classified as "decreased GFR"? a) 120 mL/min b) 70 mL/min c) 100 mL/min d) 40 mL/min

70 mL/min Normal GFR is 120-130 mL/min. Persons with a GFR of 60-89 mL/min who have no discernible renal damage are classified as "decreased GFR." Causes for this condition include removal of one kidney, fluid volume depletion, and advanced age. GFR below 60 mL/min for greater than three months is defined as chronic kidney disease

Urine is a amber, light-yellow fluid that is 5% dissolved solid. What percent of it is water?

95 Explanation: Urine is 95% water and 5% dissolved solids.

A patient has prerenal failure. The nurse knows that this type of failure is characterized by which relationship of blood urea nitrogen (BUN) to serum creatinine levels? a) An elevated BUN level and decreased creatinine level b) A BUN to creatinine level ratio of 20:1 c) A BUN to creatinine level ratio of 10:1 d) An elevated creatinine level and decreased BUN level

A BUN to creatinine level ratio of 20:1 In prerenal failure, glomerular filtration rate (GFR) decreases, allowing more filtered urea to be reabsorbed into the circulatory system. Creatinine is filtered but remains in the forming urine. Thus the BUN to creatinine ratio rises to 20:1. A ratio of 10:1 is normal.

The primary care provider for a newly admitted hospital client has added the glomerular filtration rate (GFR) to the blood work scheduled for this morning. The client's GFR results return as 50 mL/minute/1.73 m2. The nurse explains to the client that this result represents: a) Concentrated urine b) A loss of over half the client's normal kidney function c) A need to increase water intake d) The kidneys are functioning normally

A loss of over half the client's normal kidney function In clinical practice, GFR is usually estimated using the serum creatinine concentration. A GFR below 60 mL/minute/1.73 m2 represents a loss of one half or more of the level of normal adult kidney function. -The GFR is not diagnostic for concentrated urine or the need to drink more water

Which of the following statements about the use of angiotensin-converting enzyme inhibitor medications and autosomal recessive polycystic kidney disease (ARPKD) is accurate? a) ACE inhibitors should be used strictly in those clients who also have an underlying cardiac history. b) The use of ACE inhibitors will increase the vasopressin levels. c) The ACE inhibitors have been shown to shrink the size of the cysts inside the kidneys. d) ACE inhibitors may interrupt the renin-angiotensin-aldosterone system to reduce renal vasoconstriction.

ACE inhibitors may interrupt the renin-angiotensin-aldosterone system to reduce renal vasoconstriction. In addition to increasing water intake to decrease vasopressin levels, the angiotensin-converting enzyme (ACE) inhibitors or angiotensin II receptor blockers (ARBs) may be used to interrupt the renin-angiotensin-aldosterone system as a means of reducing intraglomerular pressure and renal vasoconstriction. Although not approved by the Food and Drug Administration (FDA), there has been recent interest in the use of vasopressin receptor antagonists (vaptans) to decrease cyst development.

A patient is being evaluated for kidney stones. The nurse anticipates the patient will manifest:

Acute onset of colicky or dull and achy intermittent flank pain One of the major manifestations of kidney stones is pain. Depending on the location, there are two types of pain associated with kidney stones—colicky or non colicky. The pain may radiate to the lower abdominal quadrant, bladder area, perineum, or scrotum in men. Stones are not externally visible or palpable. Obstruction by a stone may result in urinary retention and increased fluid volume.

A nurse is caring for a patient with diabetic glomerulosclerosis. The analysis is reviewed for the presence of which of the following manifestations? a) Albumin b) Red blood cells c) Sodium d) Potassium

Albumin The clinical manifestations of diabetic glomerulosclerosis are closely linked to those of diabetes. The increased glomerular filtration rate that occurs in people with early alterations in renal function is associated with microalbuminuria, which is defined as urinary albumin excretion of 30 to 300 mg in 24 hours.

A client has been given the diagnosis of diffuse glomerulonephritis. They ask the nurse what diffuse means. The nurse responds: a) All glomeruli and all parts of the glomeruli are involved. b) Only some of the glomeruli are affected. c) That the mesangial cells are being affected. d) Only one segment of each glomerulus is involved.

All glomeruli and all parts of the glomeruli are involved. Glomerular changes can be diffuse, involving all glomeruli and all parts of the glomeruli; focal, meaning only some of the glomeruli are affected; segmental, involving only a certain segment of each glomerulus; and mesangial, affecting only mesangial cells.

A client who has developed stage 3 renal failure has been diagnosed with high phosphate levels. To avoid the development of osteodystrophy, the physician may prescribe a phosphate-binding agent that does not contain: a) Aluminum b) Sevelamer hydrochloride c) Calcium acetate d) Calcium carbonate

Aluminum Aluminum-containing antacids can contribute to the development of osteodystrophy, whereas calcium-containing phosphate binders can lead to hypercalcemia, thus worsening soft tissue calcification, especially in persons receiving vitamin D therapy. Sevelamer hydrochloride is a newer phosphate-binding agent that does not contain calcium or aluminum.

Following an automobile accident where the patient had a traumatic amputation of their lower leg and lost >40% of their blood volume, they are currently not producing any urine output. The nurse bases this phenomena on which of the following humoral substances responsible for causing severe vasoconstriction of the renal vessels? a) Angiotensin II and ADH. b) Renin and potassium ions. c) Aquaproin-2 channels. d) Albumin and norepinephrine.

Angiotensin II and ADH. Increased sympathetic activity causes constriction of the afferent and efferent arterioles and thus a creases in renal blood flow. Intense sympathetic stimulation can produce marked decreases in renal blood flow and GFR. Humoral substances, including angiotensin II, ADH, and endothelins, produce vasoconstriction of renal blood flow. Aquaporin-2 channels, potassium ions, and albumin do not have vasoconstriction properties.

Which additional physical finding would you anticipate seeing in a child suspected of having a Wilms tumor? a) A visible fine red rash on the body b) Presence of a palpable flank mass c) Anomalies of the genitourinary system d) Massive generalized edema

Anomalies of the genitourinary system An important feature of Wilms tumor is its association with other congenital anomalies, aniridia (absence of the iris), hemihypertrophy (enlargement of one side of the face or body), and other congenital anomalies, usually of the genitourinary system. Wilms tumor usually is a solitary mass that occurs in any part of the kidney. The common presenting signs are a large asymptomatic abdominal mass and hypertension.

Which of the following types of pharmacological therapy does the nurse anticipate administering to a patient for treatment of a spastic bladder in order to decrease bladder hyperactivity? a) Calcium channel blockers b) Antibiotics c) Biofeedback d) Anticholinergic medications

Anticholinergic medications Correct Explanation: Among the methods used to treat spastic bladder and detrusor-sphincter dyssynergia are the administration of anticholinergic medications to decrease bladder hyperactivity and urinary catheterization to produce bladder emptying. Antibiotics are used to treat urinary tract infections. Biofeedback is a behavioral method of treatment. Calcium channel blockers would not be effective in treating this disorder.

Atelectasis is the term used to designate an incomplete expansion of a portion of the lung. Depending on the size of the collapsed area and the type of atelectasis occurring, the nurse may see a shift of the mediastinum and trachea. Which way does the mediastinum and trachea shift in compression atelectasis? a) Away from the affected lung b) Toward the affected lung c) Away from the trachea d) Toward the mediastinum

Away from the affected lung Explanation: If the collapsed area is large, the mediastinum and trachea shift to the affected side. In compression atelectasis, the mediastinum shifts away from the affected lung. None of the other answers are correct.

The nurse is caring for a patient who has had acute blood loss from ruptured esophageal varices. Which of the following does the nurse recognize is an early sign of prerenal failure? a) Baseline blood pressure of 150/90 mm Hg that is now 130/80 mm Hg b) Baseline heart rate of 100 bpm that has increased to 120 bpm c) Foul smelling, cloudy urine d) Baseline urine output of 50 mL/hr that is now 10 mL/hr

Baseline urine output of 50 mL/hr that is now 10 mL/hr The kidney normally responds to a decrease in the glomerular filtration rate with a decrease in urine output. Thus, an early sign of prerenal failure is a sharp decrease in urine output.

Which clinical manifestations would you expect to see in an infant diagnosed with autosomal recessive polycystic kidney disease (ARPKD)? a) Gross hematuria and massive generalized edema b) Elevated systemic blood pressure and severe pain c) Bilateral flank masses and impaired lung development d) Ascending urinary tract infection and vomiting

Bilateral flank masses and impaired lung development Clinical manifestations of the typical infant with ARPKD include bilateral flank masses, accompanied by severe renal failure, signs of impaired lung development, and variable degrees of liver fibrosis and portal hypertension. Potter facies and other defects associated with oligohydramnios may be present. Hypertension is usually noted within the first few weeks of life and is often severe.

A nurse is caring for a patient with systemic lupus erythematosis (SLE). The patient asks why a urinalysis is necessary. The best answer is that a urinalysis would determine whether which of the following factors was present in the urine? a) Sugar b) Blood or protein c) Cancer cells d) Potassium

Blood or protein Because of the high risk of kidney disease, all people with SLE should undergo routine urinalysis to monitor for the presence of hematuria or proteinuria.

A patient who has had an intestinal bypass has developed a kidney stone. Which of the following types of kidney stone does the nurse recognize that this patient will most likely be treated for? a) Cystine b) Uric acid c) Calcium d) Magnesium ammonium

Calcium Most kidney stones are calcium stones-calcium oxalate, calcium phosphate, or a combination of the two materials. Patients who have had intestinal bypass surgery are at a higher risk for developing calcium kidney stones.

What is the most common gram-negative bladder infection found in hospitalized clients? a) Stone formation b) Catheter-induced infection c) Neurogenic bladder d) Urinary incontinence

Catheter-induced infection Correct Explanation: Urinary catheters are a source of urethral irritation and provide a means for entry of microorganisms into the urinary tract. Catheter-associated bacteriuria remains the most frequent cause of gram-negative septicemia in hospitalized clients. Studies have shown that bacteria adhere to the surface of the catheter and initiate the growth of a biofilm that then covers the surface of the catheter.

The nurse determines that the client has clubbing of the fingertips. Which is the best intervention? a) Monitor the client's heart rate. b) Check the patient's O2 saturation level. c) Assess peripheral capillary refill. d) Call the health care provider.

Check the patient's O2 saturation level. Explanation: Clubbing of the nails indicates long-term hypoxia and should be evaluated.

A patient is diagnosed with pulmonary embolism. Which of the following symptoms would most likely be present? a) Cough and crackles b) Chest pain and dyspnea c) Left arm pain and diaphoresis d) Shallow respirations and wheezing

Chest pain and dyspnea Explanation: The manifestations of pulmonary embolism depend on the size and location of the obstruction. Chest pain, dyspnea, and increased respiratory rate are the most frequent signs and symptoms of pulmonary embolism

Bronchiectasis is considered a secondary COPD, and, with the advent of antibiotics, it is not a common disease entity. In the past, bronchiectasis often followed specific diseases. Which disease did it not follow? a) Necrotizing bacterial pneumonia b) Influenza c) Chickenpox d) Complicated measles

Chickenpox Explanation: In the past, bronchiectasis often followed a necrotizing bacterial pneumonia that frequently complicated measles, pertussis, or influenza. Chickenpox has never been linked to bronchiectasis.

Chronic obstructive pulmonary disease (COPD) is a combination of disease processes. What disease processes have been identified as being part of COPD? a) Chronic bronchitis and emphysema b) Chronic obstructive bronchitis and asthma c) Chronic obstructive bronchitis and emphysema d) Emphysema and asthma

Chronic obstructive bronchitis and emphysema Explanation: The term chronic obstructive pulmonary disease encompasses two types of obstructive airway disease: emphysema, with enlargement of air spaces and destruction of lung tissue, and chronic obstructive bronchitis, with increased mucus production, obstruction of small airways, and a chronic productive cough. Persons with COPD often have overlapping features of both disorders. Asthma and chronic bronchitis have not been identified as components of COPD.

A hospital client with a diagnosis of chronic renal failure has orders for measurement of her serum electrolyte levels three times per week. Which of the following statements best captures the relationship between renal failure and sodium regulation? a) Clients with renal failure often maintain high sodium levels because of decreased excretion. b) Clients with advanced renal failure are prone to hyponatremia because of impaired tubular reabsorption. c) Renal clients often require a sodium-restricted diet to minimize the excretion load on remaining nephrons. d) Restricting sodium intake helps to preserve nephron function and has the additional benefit of lowering blood pressure.

Clients with advanced renal failure are prone to hyponatremia because of impaired tubular reabsorption. The compromised ability of the tubular nephrons to reabsorb sodium predisposes renal clients to low serum sodium levels. A sodium restriction is thus not normally indicated.

A client has developed chronic hypoxia and has developed pulmonary hypertension (HTN). The nurse recognizes that the most likely cause of pulmonary hypertension would be: a) Decreased vascular resistance in the pulmonary vessels b) Constriction of the pulmonary vessels in response to hypoxemia c) Hardening of the pulmonary vessels due to increased fat deposits d) Constant dilation of the pulmonary vessels in response to hypoxia

Constriction of the pulmonary vessels in response to hypoxemia Explanation: Pulmonary HTN occurs as a result of chronic hypoxia. In response to hypoxia, the pulmonary vessels constrict. The pulmonary vessels differ from the systemic circulation vessels, which dilate in response to hypoxia and hypercapnia. Smooth muscle hypertrophy and proliferation of the vessel intima occur in pulmonary HTN.

A client with a history of chronic pyelonephritis has been admitted several times with recurrent bacterial infection of the urinary tract. The nurse should anticipate educating this client with regard to which common treatment regimen? a) Increase intake of cranberry juice to 2 L/day. b) Continue taking antibiotics for full 10 to 14 days even if symptoms of infection disappear. c) Take prescribed diuretics early in the day to avoid having to get up during the night. d) Force micturition every 2 hours while awake.

Continue taking antibiotics for full 10 to 14 days even if symptoms of infection disappear. Chronic pyelonephritis involves a recurrent or persistent bacterial infection superimposed on urinary tract obstruction, urine reflux, or both. Chronic obstructive pyelonephritis can be bilateral, caused by conditions that obstruct bladder outflow; or unilateral, such as occurs with ureteral obstruction. Cranberry juice, forced micturition, and diuretics are not standard treatments for chronic pyelonephritis.

The nurse is caring for a client with a condition of deficiency of antidiuretic hormone (ADH). When assessing the client, which of these findings does the nurse anticipate? a) Retention of sodium b) Low blood pressure c) Excessive urine output d) Retention of chloride

Excessive urine output Explanation: ADH regulates the ability of the kidneys to concentrate urine. When ADH is present, the water that moved from the blood into the urine filtrate in the glomeruli is returned to the circulatory system, and when ADH is absent, the water is excreted in the urine. Pathologically, deficiency of ADH leads to polyuria and dehydration

Which of the following best describes nephronophthisis-medullary cystic kidney disease? a) Small elongated cysts form in the collecting ducts and maintain contact with the nephron of origin. b) Cysts are restricted to the corticomedullary border. c) Cysts develop in the kidney as a consequence of aging, dialysis, or other conditions that affect tubular function. d) The tubule wall, which is lined by a single layer of tubular cells, expands and then rapidly closes the cyst off from the tubule of origin.

Cysts are restricted to the corticomedullary border. Nephronophthisis and adult-onset medullary cystic kidney disease both produce progressive medullary tubulointerstitial cystic disease. The other conditions described are elsewhere in the renal space.

Which physiologic change in the elderly population contributes to urinary incontinence? a) Increased bladder contractility b) Side effects of medication c) Decline in detrusor muscle function d) Increased bladder capacity

Decline in detrusor muscle function Correct Explanation: In the elderly population, overall bladder capacity is reduced as is urethral closing pressure. Changes associated with aging include decline in detrusor muscle function, decrease in bladder contraction, and impairment in bladder emptying. Medications may contribute to incontinence; however, it is not a physiologic change; it is a chemical influence.

A client has just been admitted to the emergency department after sustaining severe injuries and massive blood loss following a motor vehicle accident. The nurse predicts that the client's glomerular filtration rate will: a) Increase b) Decrease c) Remain unchanged d) Stop filtering

Decrease Explanation: Although nearly all the blood that enters the kidney flows through the cortex, less than 10% passes into the medulla and only about 1% moves into the papillae. Under conditions of decreased perfusion or increased sympathetic nervous system stimulation, blood flow is redistributed away from the cortex toward the medulla. This redistribution of blood flow decreases glomerular filtration while maintaining the urine-concentrating ability of the kidneys, a factor that is important during conditions such as shock. With decreasing flow, the filtration rate will decrease to adapt perfusion and maintain function

As chronic kidney disease progresses, the second stage (renal insufficiency) is identified by: a) Decrease in GFR to 30 to 59 mL/minute/1.73 m2 b) Diminished GFR to less than 15 mL/minute/1.73 m2 c) Decrease in GFR of 60 to 89 mL/minute/1.73 m2 d) GFR decrease to 15 to 29 mL/minute/1.73 m2

Decrease in GFR of 60 to 89 mL/minute/1.73 m2 Diminished renal reserve is characteristic of renal insufficiency, when labs remain normal but there is renal insufficiency. Only the second stage, formerly known as renal insufficiency, is characterized by a decrease in GFR of 60 to 89 mL/minute/1.73 m2. The other choices represent stage 3, 4, and 5, respectfully.

While studying about the process of urination, the nursing student learns that which of the following is known as the "muscle of micturition"? a) Trigone muscle b) External sphincter muscle c) Internal sphincter muscle d) Detrusor muscle

Detrusor muscle Explanation: The detrusor muscle is the uncle of micturition. In the bladder neck is the continuation of the detrusor muscle known as the internal urethral sphincter, which acts as a sphincter when closed. The external sphincter muscle surrounds the urethra distal to the base of the bladder. There is not a trigone muscle; the trigone is the smooth triangular area that is bounded by the openings for the both ureters and the urethra.

The nurse is instructing a patient with advanced kidney disease (AKD) about a dietary regimen. Which of the following restrictions should the nurse be sure to include in the treatment plan to decrease the progress of renal impairment in people with AKD? a) Dietary protein b) Fats c) Foods high in calcium d) Carbohydrates

Dietary protein Restriction of dietary proteins may decrease the progress of renal impairment in people with advanced renal disease. Proteins are broken down to form nitrogenous wastes, and reducing the amount of protein in the diet lowers the blood urea nitrogen and reduces symptoms.

A 1-year-old baby boy with renal dysplasia risks end-stage renal disease unless intervention occurs. Which of the following treatment options is his care team most likely to reject? a) Dietary restriction plus erythropoietin b) Renal transplantation c) Continuous cyclic peritoneal dialysis d) Continuous ambulatory peritoneal dialysis

Dietary restriction plus erythropoietin Renal transplantation and dialysis are recommended for children; of these, transplantation is the preferred treatment. Conservative measures are inappropriate in this age group because of the importance of fostering proper bone growth, especially in the first two years, and appropriate cognitive development, which is at risk due to issues such as uremic encephalopathy and the effect of renal failure upon the central nervous system of the developing child.

A client with chronic kidney disease (CKD) has developed asterixis. The nurse knows that asterixis is which of the following? a) Burning sensation in feet b) Dorsiflexion of hands and feet c) Demyelination of nerve fibers d) Unsteady gait

Dorsiflexion of hands and feet Asterixis, which is involuntary dorsiflexion of hands and feet, can develop as CKD worsens. -Burning sensation in feet, unsteady gait and demyelination of nerves can also accompany CKD, but are not part of asterixis.

Following the diagnosis of acute renal failure, the nurse knows that one of the earliest manifestations of residual tubular damage is which of the following lab/diagnostic results? a) Elevated blood urea nitrogen (BUN). b) Serum creatinine elevation. c) Inability to concentrate urine. d) Reduced glomerular filtration rate.

Elevated blood urea nitrogen (BUN). Urine tests that measure urine osmolality, urinary sodium concentration, and fractional excretion of sodium help differentiate prerenal azotemia, in which the reabsorptive capacity of the tubular cells is maintained, from tubular necrosis, in which these functions are lost. One of the earliest manifestations of tubular damage is the inability to concentrate urine. Conventional markers of serum creatinine and urea nitrogen, fractional secretion of sodium to assess glomerular filtration rate (GFR), and urine output do not manifest for 1-2 days after the acute renal failure has begun.

Which laboratory findings should you expect to see in a patient diagnosed with nephritic syndrome? a) Decreased high-density lipoproteins and increased iron levels b) Low triglycerides and elevated sodium levels c) Elevated urine protein and hypoalbuminemia d) Abnormal blood clotting and elevated blood pressure

Elevated urine protein and hypoalbuminemia The nephritic syndrome is due to glomerular disease that is usually of acute onset and is accompanied by grossly visible hematuria, mild to moderate proteinuria, and hypertension. The nephrotic syndrome, also due to glomerular disease, is characterized by heavy proteinuria, hypoalbuminemia, and severe edema.

The nurse is caring for a client with suspected dehydration. Which of these results does the nurse recognize will help confirm this diagnosis? a) White blood cells in the urine b) Elevated urine specific gravity c) Blood in the urine d) Casts in the urine

Elevated urine specific gravity Explanation: Gravity of 1.030 to 1.040. During periods of marked hydration, the specific gravity can approach 1.000. With diminished renal function, there is a loss of renal concentrating ability, and the urine specific gravity may fall to levels of 1.006 to 1.010 (usual range is 1.010 to 1.025 with normal fluid intake).

Which of the following is the most common cause of lower uncomplicated urinary tract infections? a) H. pylori b) Proteus amoeba c) Escherichia coli d) Helicobacter

Escherichia coli Correct Explanation: Most uncomplicated lower UTIs are caused by Escherichia coli. Other common uropathic pathogens include Enterococcus faecalis, Staphylococcus saprophyticus, Klebsiella pneumoniae, Proteus mirabilis, and Pseudomonas species. Helicobacter, H. pylori, and Proteus amoeba do not cause UTIs.

Urine specific gravity is normally 1.010 to 1.025 with adequate hydration. When there is loss of renal concentrating ability due to impaired renal function, low concentration levels are exhibited. When would the nurse consider the low levels of concentration to be significant? a) Last void at night b) First void in morning c) After a nap d) At noon

First void in morning Explanation: With diminished renal function, there is a loss of renal concentrating ability, and the urine specific gravity may fall to levels of 1.006 to 1.010 (usual range is 1.010 to 1.025 with normal fluid intake). These low levels are particularly significant if they occur during periods that follow a decrease in water intake (e.g., during the first urine specimen on arising in the morning). The other answers are incorrect

The nurse is caring for a client with a disease causing excess antidiuretic hormone. When performing the assessment, the nurse should focus on which of these consequences of excess ADH? a) Fluid volume excess b) Excess salivation c) Dehydration d) Increased urinary output

Fluid volume excess Explanation: When ADH is present, the water that moved from the blood into the urine filtrate in the glomeruli is returned to the circulatory system, and when ADH is absent, the water is excreted in the urine, causing fluid retention or fluid volume excess if pathology is present.

A patient has passed a kidney stone composed of uric acid (urate). Which of the following pathological conditions is a contributing factor for the development of this type of kidney stone? a) Arteriosclerosis b) Atherosclerosis c) Gout d) Sickle-cell disease

Gout Uric acid stones develop in conditions of gout and high concentrations of uric acid in the urine.

Acute postinfectious glomerulonephritis, as its name implies, follows an acute infection somewhere else in the body. What is the most common cause of acute postinfectious glomerulonephritis? a) E. coli b) S. aureus c) Group A Β-hemolytic streptococci d) P. aeruginosa

Group A Β-hemolytic streptococci Group A Β-hemolytic streptococci have the ability to seed from one area of the body to another. One area it seeds to is the kidney, where it causes acute postinfectious glomerulonephritis. Other organisms can cause acute postinfectious glomerulonephritis, but they are not the most common cause of the disease.

A nurse is caring for a patient who has a recent history of passing calcium urinary stones. Which of the following is a priority nursing consideration for this patient? a) Bowel elimination b) Skin care c) Bed rest d) Hydration

Hydration A major goal of treatment in people who have passed kidney stones or have had them removed is to prevent their recurrence. Adequate fluid intake reduces the concentration of stone-forming crystals in the urine and needs to be encouraged.

A nurse is assessing a client for early manifestations of chronic kidney disease (CKD). Which would the nurse expect the client to display? a) Terry nails b) Hypertension c) Asterixis d) Impotence

Hypertension Hypertension is commonly an early manifestation of CKD. The mechanisms that cause the hypertension are multifactorial; they include increased vascular volume, increased peripheral vascular resistance, decreased levels of renal vasodilator prostaglandins, and increase activity of the renin-angiotensin-aldosterone system. -Impotence occurs in as many as 56% of males on dialysis. -Terry nails are dark band just behind the leading edge of a fingernail followed by a white band that occur in the late stages. -Asterixis, a sign of hepatic encephalopathy, is due to the inability of the liver to metabolize ammonia to urea.

The nurse is caring for a client diagnosed with pneumonia. The client's arterial blood gas results identify decreased level oxygen in the arterial blood and a decreased in the partial pressure of oxygen. The nurse would interpret this as: a) Hypercapnia b) Hypoxemia c) Hypoxia d) Atelectasis

Hypoxemia Explanation: Hypoxemia refers to a reduction in PO2 (partial pressure of oxygen) of the arterial blood. -Hypoxemia can result from an inadequate amount of O2 in the air, disease of the respiratory system, dysfunction of the neurologic system, or alterations in circulatory function. - Hypercapnia is an increase in the carbon dioxide content of the arterial blood. -Atelectasis is collapse of a segment of the lung due to airway obstruction.

The patient who has been admitted with a problem with his bladder has a postvoid residual (PVR) of 250 ml. The nurse understands that this indicates which of the following? a) Inadequate bladder emptying b) Adequate bladder emptying c) No problem at all d) A problem with storage of urine

Inadequate bladder emptying Correct Explanation: Postvoid residual (PVR) urine volume provides information about bladder emptying and NOT storage. A PVR value of less than 50ml is considered adequate bladder emptying and more than 200 ml indicates inadequate bladder emptying.

A client reports that she A client reports that she frequently suffers from UTIs after engaging in sexual intercourse. Which would be the best information for the nurse to provide? a) Refrain from intercourse b) Delay voiding for 5 hours after intercourse c) Decrease fluid intake before intercourse d) Increase fluid intake before intercourse

Increase fluid intake before intercourse Correct Explanation: A nonpharmacologic approach to the treatment of frequent UTIs associated with sexual intercourse is to increase fluid intake before intercourse and to void soon after intercourse. This procedure uses the washout phenomenon to remove bacteria from the bladder.

The nurse is assessing a patient who has a unilateral obstruction of the urinary tract. The nurse anticipates the patient may develop: a) Excretion of dilute urine b) Increase in blood pressure c) Inability to control urination d) Increased urine output

Increase in blood pressure Hypertension is an occasional complication of urinary tract obstruction. It is more common in cases of unilateral obstruction in which renin secretion is enhanced, probably secondary to impaired renal blood flow. In these circumstances, removal of the obstruction often leads to a reduction in blood pressure. The urine output would be decreased and not diluted

When teaching the client with gout about the cause of the disease, which of these should the nurse relate? a) Retention of sodium can cause gout. b) Increased levels of uric acid in the blood cause gout. c) Urea, in the form of blood urea nitrogen, causes gout. d) High blood pressure causes gout.

Increased levels of uric acid in the blood cause gout. Explanation: Uric acid is a product of purine metabolism. High blood levels of uric acid (hyperuricemia) can cause gout, and excessive urine levels can cause kidney stones.

Which of the following data would a clinician consider as most indicative of acute renal failure? a) Decreased serum creatinine and blood urea nitrogen (BUN); decreased potassium and calcium levels b) Decreased urine output; hematuria; increased GFR c) Alterations in blood pH; peripheral edema d) Increased nitrogenous waste levels; decreased glomerular filtration rate (GFR).

Increased nitrogenous waste levels; decreased glomerular filtration rate (GFR). The hallmark of acute renal injury is azotemia, an accumulation of nitrogenous wastes such as creatinine, urea nitrogen, and uric acid plus a decrease in the GFR of the kidneys. While pH alterations, edema, electrolyte imbalances and decreased urine output may accompany acute renal failure, they are all potentially attributable to other pathologies. Creatinine, GFR and BUN would be unlikely to rise during renal failure.

A patient is to receive a radiocontrast media as part of a diagnostic scan. Which of the following is intended to reduce the nephrotoxic effects of the radiocontrast media? a) Administering ibuprophen 600 mg prior to the procedure b) Administering one unit of packed red blood cells c) Having the patient take nothing by mouth d) Increasing the normal saline intravenous infusion rate to 125 mL/hour

Increasing the normal saline intravenous infusion rate to 125 mL/hour Some drugs, such as high-molecular-weight radiocontrast media, the immunosuppressive drugs cyclosporine and tacrolimus and nonsteroidal anti-inflammatory drugs can cause acute prerenal failure by decreasing renal blood flow. Administering intravenous saline can improve hydration and renal perfusion to decrease the toxic effects of the radiocontrast media.

Pleuritis, an inflammatory process of the pleura, is common in infectious processes that spread to the pleura. Which are the drugs of choice for treating pleural pain? a) Indomethacin b) Inderal c) Warfarin d) Acetaminophen

Indomethacin Explanation: Treatment of pleuritis consists of treating the underlying disease and inflammation. Analgesics and nonsteroidal anti-inflammatory drugs (e.g., indomethacin) may be used for pleural pain. Although these agents reduce inflammation, they may not entirely relieve the discomfort associated with deep breathing and coughing. The other answers are not used to treat pleural pain

Which one of the following is the usual cause of acute pyelonephritis? a) Autoimmunity b) Obstruction c) Infection d) Reflux

Infection Gram-negative bacteria, including Escherichia coli and Proteus, Klebsiella, Enterobacter, and Pseudomonas species, are the most common causative agents for acute pyelonephritis.

A client has acute pyelonephritis. The nurse will monitor the client for development of which of the following? a) Prerenal failure b) Post renal failure c) Intrarenal failure d) Chronic kidney disease

Intrarenal failure Acute pyelonephritis, an active bacterial infection, can cause tubular cell necrosis and intrarenal failure. Acute pyelonephritis does not cause prerenal or postrenal failure or chronic kidney disease unless it is not treated

A female client asks the nurse if there is any noninvasive treatment to help with the involuntary loss of urine that occurs when she coughs or sneezes. Which is the best response by the nurse? a) Self-catheterization b) Periurethral injection of a bulking agent c) Implanted artificial sphincter d) Kegel exercises

Kegel exercises Kegel exercise involves repetitive contraction and relaxation of the pelvic floor muscles and is a noninvasive way of strengthening the pelvic floor muscles and is a client-dependent behavioral intervention. The remaining options are invasive procedures.

A patient with stress incontinence states, "every time I laugh or cough, I urinate on myself. Which of the following behavioral measures would be beneficial for the nurse to teach the patient? a) Administration of tricyclic antidepressants b) Care of an indwelling Foley catheter c) Kegel exercises d) Administration of anticholinergic medications

Kegel exercises Behavioral methods for incontinence include fluid management, timed/prompted voiding, pelvic floor exercises (Kegel exercises), bladder retraining, and toileting assistance. Bladder retraining and biofeedback techniques seek to reestablish cortical control over bladder function by having the person ignore urgency and respond only to cortical signals during waking hours. Indwelling catheters are not used routinely to control incontinence and is a medical intervention. Administering medications is considered to be a pharmacological intervention.

Vitamin D metabolism is deranged in clients with chronic kidney disease (CKD). The nurse recognizes that which of the following statements regarding vitamin D is correct? a) Calcitriol stimulates release of parathyroid hormone (PTH). b) Calcitriol blocks gastrointestinal absorption of calcium. c) Suppression of parathyroid hormone release is characteristic of CKD. d) Kidneys convert inactive vitamin D to its active form, calcitriol.

Kidneys convert inactive vitamin D to its active form, calcitriol. Inactive vitamin D is converted to active calcitriol in the kidneys. Calcitriol enhances gastrointestinal absorption of calcium, and suppresses release of PTH. Elevated levels of PTH are characteristic of CKD as blood levels of phosphate rise and levels of calcium fall.

Staghorn kidney stones, or struvite stones, are usually located in the renal pelvis. These stones are made from what? a) Uric acid b) Calcium oxalate c) Magnesium ammonium phosphate d) Cystine

Magnesium ammonium phosphate Phosphate levels are increased in alkaline urine and magnesium, always present in the urine, and combine to form struvite stones. These stones can increase in size until they fill an entire renal pelvis. Because of their shape, they often are called staghorn stones. The other minerals can form stones, but not staghorn stones.

A patient has a marked decrease in the glomerular filtration rate (GFR) and has elevations in blood urea nitrogen, potassium, and creatinine levels. The patient's urine output has decreased to 10 mL/hr and has 3+ pitting edema in the lower extremities. In which of the following phases of acute tubular necrosis does the nurse determine the patient is in relating to this assessment data? a) Maintenance phase b) Onset phase c) Initiating phase d) Recovery phase

Maintenance phase The maintenance phase of acute tubular necrosis is characterized by a marked decrease in the GFR causing sudden retention of endogenous metabolites—such as urea, potassium, sulfate, and creatinine—that normally are cleared by the kidneys. Urine output usually is lowest at this point. Fluid retention gives rise to edema, water intoxication, and pulmonary congestion. If the period of oliguria is prolonged, hypertension frequently develops and with it signs of uremia.

A nurse is caring for a patient who is diagnosed with acute poststreptococcal glomerulonephritis. Which of the following is the most common clinical presentation of this condition? a) Occult hematuria b) Nephrotic syndrome c) Nephritic syndrome d) Renal colic

Nephritic syndrome The acute nephritic syndrome is the clinical correlate of acute glomerular inflammation. The nephritic syndromes produce a proliferative inflammatory response, whereas the nephrotic syndrome produces increased permeability of the glomerulus. The acute nephritic syndrome may occur in such a systemic disease as systemic lupus erythematosus. Typically, however, it is associated with acute proliferative glomerulonephritis such as postinfectious glomerulonephritis. Acute postinfectious glomerulonephritis usually occurs after infection with certain strains of A beta-hemolytic streptococci and is caused by deposition of immune complexes of antibody and bacterial antigens

The health care provider has prescribed an aminoglycoside (gentamicin) for a client. The nurse is aware that the client is at risk for: a) Ischemic acute tubular necrosis b) Nephrotoxic acute tubular necrosis c) Postrenal failure d) Chronic kidney disease

Nephrotoxic acute tubular necrosis Pharmacologic agents that are directly toxic to the renal tubule include aminoglycosides (e.g., gentamicin), cancer chemotherapeutic agents such as cystplastin and ifosfamide, and radiocontrast agents. Nephrotoxic agents cause tubular injury by inducing varying combinations of renal vasoconstriction, direct tubular damage, or intratubular obstruction. -Postrenal failure results from obstruction of outflow of the kidneys. -CKD and its treatment can interfere with the absorption, distribution, and elimination of drugs. -Acute tubular necrosis (ATN) occurs most frequently in clients who have major trauma, severe hypovolemia, overwhelming sepsis, trauma, or burns.

A 45-year-old client with chronic kidney disease (CKD) voices concern about her dialysis treatment. The client would like to work and spend time with her family. Which type of dialysis will best fit this client's lifestyle? a) Hemodialysis b) Continuous ambulatory peritoneal dialysis c) Nocturnal intermittent peritoneal dialysis (NIPD) d) Continuous cyclic peritoneal dialysis

Nocturnal intermittent peritoneal dialysis (NIPD) In NIPD, the client is given 10 hours of automatic cycling each night, with the abdomen left dry during the day. This is the most beneficial for this client. Individual preference, manual ability, lifestyle, knowledge of the procedure, and physiologic response to treatment are used to determine the type of dialysis that is used. Hemodialysis is generally three times a week for 3 to 4 hours a day. CAPD involves exchanging the dialysate four to six times per day. In CCPD, the last exchange remains in the abdomen during the day

A drug abuser was found unconscious after shooting up heroin 2 days prior. Because of the pressure placed on the hip and arm, the client has developed rhabdomyolysis. The nurse knows this can: a) Be cured by administering an anticoagulant immediately b) Cause the kidney to develop renal stones due to stasis c) Cause compartment syndrome in the lower extremities d) Obstruct the renal tubules with myoglobin and damage tubular cells

Obstruct the renal tubules with myoglobin and damage tubular cells Myoglobin normally is not found in the serum or urine. It has a low molecular weight; if it escapes into the circulation, it is rapidly filtered in the glomerulus. A life-threatening condition known as rhabdomyolysis occurs when increasing myoglobinuria levels cause myoglobin to precipitate in the renal tubules, leading to obstruction and damage to surrounding tubular cells. Myoglobinuria most commonly results from muscle trauma but may result from exertion, hyperthermia, sepsis, prolonged seizures, and alcoholism or drug abuse. -Rhabdomyolysis is not cured with anticoagulation administration nor does it cause kidney stones. -Compartment syndrome occurs when there is insufficient blood supply to muscles and nerves due to increased pressure within one of the body's compartments.

Surgical stone removal is indicated for which of the following instances? a) Stone too small to pass b) Mild, intermittent pain for one day c) Obstruction of urine flow d) Small stone without evidence of size increase

Obstruction of urine flow Open stone surgery may be required to remove large calculi or those that are resistant to other forms of removal. Kidney stones are a major cause of upper urinary tract obstruction.

A patient is concerned about the possibility of having bladder cancer after his brother was diagnosed with it 2 years ago. Which of the following assessment data obtained by the nurse would indicate that the patient should be screened for this disease? a) Patient reports diminished stream. b) Patient reports pain when he urinates. c) Patient reports that his urine has a foul smell and it is cloudy. d) Patient reports that he occasionally has blood in his urine but has no pain with it.

Patient reports that he occasionally has blood in his urine but has no pain with it. Explanation: The most common sign of bladder cancer is painless hematuria. Gross hematuria is a presenting sign in the majority of people with the disease, and microscopic hematuria is present in most others. Frequency, urgency, and dysuria occasionally accompany the hematuria. Because hematuria is often intermittent, the diagnosis may be delayed.

A nurse is caring for a patient in spinal shock. Which of the following interventions is appropriate in relation to the patient's urinary status? a) Assess bowel sounds b) Perform a urine culture c) Perform intermittent catheterization d) Palpate the lower abdomen

Perform intermittent catheterization After a spinal cord injury, the early effects on bladder function are quite different from those that follow recovery from the initial injury. During the period immediately after spinal cord injury, a state of spinal shock develops, during which all the reflexes, including the micturition reflex, are depressed. The bladder is atonic and cannot contract. Intermittent catheterization is necessary to prevent injury to urinary structures associated with overdistension of the bladder.

An 87 year-old male resident of an assisted living facility has been consistently continent of urine until the last several weeks. Which of the following actions by the care providers at the facility is the most likely priority? a) Providing client education focusing on the fact that occasional incontinence is a normal, age-related change. b) Performing a physical examination and history to determine the exact cause and character of the incontinence. c) Teaching the resident about protective pads, collection devices and medications that may be effective. d) Showing the resident the correct technique for exercises to improve bladder, sphincter and pelvic floor tone.

Performing a physical examination and history to determine the exact cause and character of the incontinence. The priority in the treatment of incontinence in the elderly is an acknowledgement that it is not an inevitability and that the exact causes should and most often can be identified. This identification by way of history-taking and examination would supersede teaching about protective devices or exercises

While assessing a peritoneal dialysis client in his or her home, the nurse notes that the fluid draining from the abdomen is cloudy, is white in color, and contains a strong odor. The nurse suspects this client has developed a serious complication known as: a) Bladder erosion b) Bowel perforation c) Peritonitis d) Too much sugar in the dialysis solution

Peritonitis Potential problems with peritoneal dialysis include infection, catheter malfunction, dehydration, hyperglycemia, and hernia. -Bowel perforation can occur, but the fluid would be stool colored. -The client may develop hyperglycemia; however, this will not cause the fluid to be cloudy. -If bladder erosion had occurred, the fluid would look like urine and not be cloudy and white.

A patient diagnosed with Goodpasture's syndrome would require which of the therapies to remove proteins and autoantibodies from the system? a) Intervenous calcium b) Renal transplant c) Kidney removal d) Plasmapheresis

Plasmapheresis Plasmapheresis is used to filter the blood for removal of proteins and the circulating anti-GBM (glioblastoma multiforme) antibodies. Renal transplantation would return the kidneys to normal function, but are the extreme of treatment. The other options would not produce the necessary treatment outcomes

Which one of the following would the nurse see as being liable to cause the most serious long-term problems? a) Unilateral renal agenesis b) Polycystic kidney disease c) Simple renal cyst d) Horseshoe kidney

Polycystic kidney disease Agenesis refers to failure of an organ to develop at all. The other kidney usually undergoes compensatory hypertrophy and performs the function of the missing kidney. Most simple cysts do not produce signs or symptoms or compromise renal function. A horseshoe kidney occurs when the upper and lower poles of the two kidneys are fused, producing a horeshoe-shaped structure. The condition usually does not cause problems. Polycystic kidneys may be associated with aneurysm, and subarachnoid hemorrhage is a frequent cause of death.

The nurse knows that one of the earliest manifestations of acute renal failure (ARF) is which of the following? a) Elevated plasma uric acid b) Polyuria c) Elevated urinary glucose d) Hypokalemia

Polyuria One of the earliest signs of ARF is loss of ability to concentrate urine. The other answer selections are not notable markers of early renal failure; however, they may develop as renal failure progresses.

A patient has an obstructive urine outflow related to benign prostatic hyperplasia. Due to the inability to excrete adequate amounts of urine, which of the following types of renal failure should the nurse closely monitor for? a) Postrenal failure b) Intrarenal failure c) Prerenal failure d) Chronic renal failure

Postrenal failure Postrenal failure results from obstruction of urine outflow from the kidneys. The obstruction can occur in the ureter, bladder, or urethra. Due to the increased urine not being able to be excreted due to the obstruction, retrograde pressure occurs throughout the tubules and nephrons, which ultimately damages the nephrons. Prostatic hyperplasia is the most common underlying problem

A patient in renal failure has marked decrease in renal blood flow caused by hypovolemia, caused by gastrointestinal bleeding. The nurse is aware that this form of renal failure can be reversed if the bleeding is under control. Which of the following forms of acute renal injury does this patient have? a) Postrenal failure b) Chronic renal failure c) Prerenal failure d) Intrarenal failure

Prerenal failure Prerenal failure, the most common form of acute renal failure, is characterized by a marked decrease in renal blood flow. It is reversible if the cause of the decreased renal blood flow can be identified and corrected before kidney damage occurs.

The nurse is reviewing the diagnosis of four male clients. Select the diagnosis that places the clients at risk for developing postrenal kidney failure. a) Severe hypovolemia b) Acute pyelonephritis c) Intratubular obstruction d) Prostatic hyperplasia

Prostatic hyperplasia The most common cause of postrenal kidney failure is prostatic hyperplasia. Postrenal failure results from conditions that obstruct urine outflow. The obstruction can occur in the ureter, bladder, or urethra. -Intratubular obstruction and acute pyelonephritis are intrarenal causes of kidney failure, and severe hypovolemia is a prerenal cause.

Which of the following substances would not be found in glomerular filtrate? a) Water b) Protein c) Potassium d) Sodium

Protein The glomerular filtrate has a chemical composition similar to plasma which contains sodium, potassium and water, but it contains no proteins because large molecules do not readily cross the glomerular wall.

The nurse is reviewing the lab results of a patient with suspected nephrotic syndrome. The nurse anticipates that the results to include: a) Abnormal blood clotting factors b) Serum hyperalbuminemia c) Protein in the urine d) Decreased tryglycerides e) Decreased low-density lipoproteins

Protein in the urine In a person with nephrotic syndrome there is massive proteinuria (protein in the urine), serum hypoalbuminemia, generalized edema and hyperlipidemia.

The nurse recognizes that acute renal injury is characterized by which of the following? a) Rapid decline in renal function b) Irreversible damage to nephrons c) Decreased blood urea nitrogen (BUN) d) Low incidence of mortality

Rapid decline in renal function Acute renal injury is a rapid decline in kidney function. BUN rises as nitrogenous wastes are not removed from the circulation. If the cause can be ameliorated, the injury is usually reversible. Most at risk are seriously ill clients; the mortality rate is between 40% and 90% in these clients.

The nurse reviews the lab results for a patient who has advanced autosomal dominant polycystic kidney disease (ADPKD). The patient 's hemoglobin is 8.8 g/dL. The nurse suspects this lab value is related to which of the following causes? a) Poor dietary intake of iron b) Low calcium levels c) Reduced production of erythropoietin d) Hemorrhage

Reduced production of erythropoietin As ADPKD progresses, the nephrons reduce the production of erythropoietin (EPO). EPO is necessary for red blood cell production by bone marrow, so EPO deficiency causes anemia.

A 72-year-old patient is scheduled for a kidney transplant. The nurse knows that which aspect of advanced age has a positive effect on the success of kidney transplant survival? a) Decreased muscle mass b) Psychological maturity c) Reduction in T-lymphocyte function d) Acceptance of immunosuppressive therapy

Reduction in T-lymphocyte function The general reduction in T-lymphocyte function with subsequent decrease in immune system activity that occurs with aging would foster transplant survival. The other options would not have a direct effect on rejection of the transplant. -Acceptance of immunosuppressive therapy is not a specific characteristic of advanced age. -Psychological maturity and decreased muscle mass have little to no effect on transplant survival.

Which of the following phenomena contributes to the difficulties with absorption, distribution and elimination of drugs that is associated with kidney disease? a) Reductions in plasma proteins increase the amount of free drug and decrease the amount of protein-bound drug. b) Acute tubular necrosis is associated with impaired drug reabsorption through the tubular epithelium. c) Decreased retention by the kidneys often renders normal drug dosages ineffective. d) Dialysis removes active metabolites from circulation minimizing therapeutic effect.

Reductions in plasma proteins increase the amount of free drug and decrease the amount of protein-bound drug. A decrease in plasma proteins, particularly albumin, that occurs in many persons with renal failure results in less protein-bound drug and greater amounts of free drug. -Drug elimination problems do not stem as directly from impaired tubular reabsorption, decreased retention or the process of dialysis.

A client is admitted with worsening heart failure. The client is complaining about having to urinate frequently. The nurse knows that the physiology behind the body's response to decrease vascular volume by increasing urine output is due to: a) Aldosterone secretion by the adrenal gland, which inhibits sodium absorption b) Release of atrial natriuretic peptide (ANP) from overstretched atria c) Renin secretion, resulting in angiotensin II formation d) Reabsorption of potassium from the proximal tubule

Release of atrial natriuretic peptide (ANP) from overstretched atria Explanation: ANP is believed to play an important role in salt and water excretion by the kidney. It is synthesized by muscle cells in the atria of the heart and released when the atria are stretched. Increased levels of this peptide directly inhibit the reabsorption of sodium and water in the renal tubules. ANP also inhibits renin secretion and therefore angiotensin II formation, which in turn reduces reabsorption of sodium. The decrease in sodium reabsorption increases urine output and helps return blood volume to normal. ANP levels, which become elevated when the atria are stretched in HF, help to decrease vascular volume by increasing urine output. Potassium reabsorption is not responsible for water excretion. Aldosterone secretion by the adrenal gland functions in the regulation of sodium and potassium elimination by the principal cells in the distal and collecting tubules.

A client who is admitted to the hospital receives a diagnosis of lower urinary tract obstruction. What does the nurse pick to be the immediate treatment for this problem? a) Bladder training b) Pain med c) Correcting the problem causing the obstruction d) Relief of bladder distention

Relief of bladder distention The immediate treatment of lower urinary tract obstruction and stasis is directed toward relief of bladder distention, which is usually accomplished through catheterization. Correcting the problem that is the cause is a long-term treatment. Pain med does not help the problem, nor does bladder training.

Which of the following is the most common cancer of the kidney? a) Lymphoma b) Renal cell carcinoma c) Wilms tumor d) Transitional cell carcinoma

Renal cell carcinoma Renal cell carcinoma accounts for 80% to 90% of all kidney tumors.

Neuromuscular disorders can be triggered by CKD. For those clients on dialysis, approximately two thirds suffer from what peripheral neuropathy? a) Raynaud syndrome b) Tingling and loss of sensation in lower limbs c) Burning hands and feet d) Restless legs syndrome

Restless legs syndrome Restless legs syndrome is a manifestation of peripheral nerve involvement and can be seen in as many as two thirds of clients on dialysis. The other answers are not correct.

A client diagnosed with chronic kidney disease (CKD) is experiencing nausea and vomiting. Which would be the best instruction for the nurse to provide? a) Restrict intake of dietary fat b) Increase intake of fruit juice c) Increase intake of carbohydrates d) Restrict intake of dietary protein

Restrict intake of dietary protein Early morning nausea is common in CKD. Nausea and vomiting often improve with the restriction of dietary protein and after initiation of dialysis and disappears after kidney transplant. The other options will not improve the symptoms.

The GFR is considered to be the best measure of renal function. What is used to estimate the GFR? a) BUN b) Serum creatinine c) Serum protein d) Albumin level

Serum creatinine In clinical practice, GFR is usually estimated using the serum creatinine concentration. The other answers are not used to estimate the GFR.

An 86-year-old female client has been admitted to the hospital for the treatment of dehydration and hyponatremia after she curtailed her fluid intake to minimize urinary incontinence. The client's admitting laboratory results are suggestive of prerenal failure. The nurse should be assessing this client for which of the following early signs of prerenal injury? a) Intermittent periods of confusion b) Excessive voiding of clear urine c) Acute hypertensive crisis d) Sharp decrease in urine output

Sharp decrease in urine output Dehydration and its consequent hypovolemia can result in acute renal failure that is prerenal in etiology. The kidney normally responds to a decrease in GFR with a decrease in urine output. Thus, an early sign of prerenal injury is a sharp decrease in urine output. Post-renal failure is obstructive in etiology, and intrinsic (or intrarenal) renal failure is reflective of deficits in the function of the kidneys themselves.

A 22 year-old female with a history of intermittent flank pain, repeated UTIs and hematuria has been diagnosed with autosomal dominant polycystic kidney disease (ADPKD). Which of the following phenomena has most likely contributed to the development of this diagnosis? a) She has inherited undersized kidneys that are prone to calculi formation. b) UTIs coupled with an impaired immune response have caused her ADPKD. c) She has inherited a tendency for epithelial cell in her tubules to proliferate inappropriately. d) Severe hypertension and portal hypertension are likely precursors.

She has inherited a tendency for epithelial cell in her tubules to proliferate inappropriately. ADPKD is an inherited condition, and the etiology is thought to involve cysts arising in segments of the renal tubules from a few epithelial cells that proliferate abnormally. UTIs are consequent, not causative, of the condition. Severe hypertension and portal hypertension are more commonly associated with ARPKD (autosomal recessive polycystic kidney disease) than ADPKD. Kidneys are typically oversized in ADPKD and renal calculi are not noted sequelae

A child is brought to the emergency department with an asthma attack. Assessment revealed the use of accessory muscles, a weak cough, audible wheezing sound, moist skin, and tachycardia. Which of the following drugs will the nurse anticipate administering first? a) Anti-inflammatory agents such as sodium cromolyn b) Oral corticosteroids c) Short-acting beta 2-adrenergic agonists such as albuterol (SABA) d) Anticholinergic medications such as ipratropium

Short-acting beta 2-adrenergic agonists such as albuterol (SABA) Explanation: The quick-relief medications such as SABA (e.g., albuterol, levalbuterol, pirbuterol) relax bronchial smooth muscle and provide prompt relief of symptoms, usually within 30 minutes. They are administered by inhalation (i.e., metered-dose inhaler or nebulizer), and their recommended use is in alleviating acute attacks of asthma.

In the intensive care unit (ICU), the nurse is caring for a trauma client who has abdominal injuries is beginning to have a decrease in BP and increased pulse rate and is pale with diaphoretic skin. The nurse is assessing the client for hemorrhagic shock. If the client is in shock, the nurse would expect to find: a) Complaints of flank pain rotating around the abdominal muscles b) Significant decrease in urine output due to decrease in renal blood flow c) Excess output of blood-tinged urine d) An increase in GFR due to relaxation of the afferent arterioles

Significant decrease in urine output due to decrease in renal blood flow Explanation: During periods of strong sympathetic stimulation, such as shock, constriction of the afferent arteriole causes a marked decrease in renal blood flow and thus glomerular filtration pressure. Consequently, urine output can fall almost to 0. Unless the injury is specific to the kidney, the client will not have blood in urine and urine production will not be excessive. Flank pain is associated with obstruction due to stone formation. The GFR will decrease rather than increase.

Drug-related nephropathies occur all too often. They involve functional and/or structural changes to the kidney after exposure to a drug. What does the tolerance to drugs depend on? a) Glomerular filtration rate b) State of hydration c) Proteinuria d) Vesicoureteral reflux

State of hydration The tolerance to drugs varies with age and depends on renal function, state of hydration, blood pressure, and the pH of the urine. None of the other answers are correct.

A nurse is caring for a patient who has been diagnosed with kidney colic and has not yet passed the stone. Which of the following interventions should the nurse be sure to include when planning the care for this patient? a) Have the client take cool baths. b) Administer acetominophen (Tylenol) every 4 hours for pain. c) Restrict fluid intake. d) Strain the urine.

Strain the urine. All urine should be strained during an attack in the hope of retrieving the stone for chemical analysis and determination of type. This information, along with a careful history and laboratory tests, provides the basis for long-term preventative measures. The patient will require an increase in fluid intake. The patient will also require an opioid analgesic to control the pain

A client who has a problem with incontinence loses a small amount of urine every time she coughs or sneezes. This type of incontinence is known as which of the following? a) Urge b) Functional c) Overflow d) Stress

Stress Stress incontinence is the involuntary loss of urine associated with activities such as coughing and sneezing. Urge incontinence is the urgency and frequency associated with hyperactivity of the detrusor muscle. Overflow incontinence is the involuntary loss of urine when intravesicular pressure exceeds maximal urethral pressure. Functional incontinence is the lack of cognitive function to go to the bathroom.

A patient is describing difficulty with urinating and informs the nurse that every time she coughs or laughs, she urinates and has begun to wear a thin pad. Which of the following types of urinary incontinence is the patient describing? a) Stress incontinence b) Urge incontinence c) Functional incontinence d) Overflow

Stress incontinence Stress incontinence is the loss of urine associated with activities, such as coughing, that increase intra-abdominal pressure. Overactive bladder/urge incontinence is urgency and frequency associated with hyperactivity of the detrusor muscle may or may not involve involuntary loss of urine. Overflow is the involuntary loss of urine when intravesicular pressure exceeds maximal urethral pressure in the absence of detrusor activity. Functional incontinence is a lack of cognitive function to go to the bathroom, commode, or urinal/bedpan resulting in spontaneous urination.

Because they strengthen the pelvic floor muscles, Kegel exercises are most likely to help: a) Mixed incontinence b) Overflow incontinence c) Stress incontinence d) Urge incontinence

Stress incontinence Correct Explanation: Stress incontinence is commonly caused by weak pelvic floor muscles, which allow the angle between the bladder and the posterior proximal urethra to change so that the bladder and urethra are positioned for voiding when some activity increases intra-abdominal pressure. Overflow incontinence results when the bladder becomes distended and detrusor activity is absent. Urge incontinence is probably related to CNS control of bladder sensation and emptying or to the smooth muscle of the bladder. Mixed incontinence, a combination of stress and urge incontinence, probably has more than one cause.

A client has experienced severe hemorrhage and is in prerenal failure. The nurse anticipates the laboratory results of the client's BUN and serum creatinine to identify: a) BUN elevates and creatinine decreases. b) Creatinine level rises and BUN decreases. c) The BUN-to-creatinine ratio is 20:1. d) The BUN-to-creatinine ratio is 10:1.

The BUN-to-creatinine ratio is 20:1. Prerenal injury is manifested by a sharp decrease in urine output and a disproportionate elevation of blood urea nitrogen (BUN) in relation to serum creatinine levels. The kidney normally responds to a decrease in the GFR with a decrease in urine output. An early sign of prerenal injury is a sharp decrease in urine output. A low GFR allows more time for small particles such as urea to be reabsorbed into the blood. Creatinine, which is larger and nondiffusible, remains in the tubular fluid, and the total amount of creatinine that is filtered, although small, is excreted in the urine. Consequently, there also is a disproportionate elevation in the ratio of BUN to serum creatinine, from a normal value of 10:1 to a ratio greater than 20:1.

A 61 year-old woman who has had an upper respiratory infection for several weeks has presented to her family physician with complaints of a recent onset of urinary retention. She reveals to her physician that she has been taking non-prescription cold medications over and above the suggested dose for the past two weeks. Which of the following phenomena will her physician most likely suspect is contributing to her urinary retention? a) Antihistamine effects inhibit communication between the pons and the thoracolumbar cord. b) The anticholinergic effects of the medication are impairing normal bladder function. c) Cholinergic actions of the cold medicine are triggering internal and external sphincter contraction. d) Over-the-counter medications such as cold medicine stimulate the parasympathetic nervous system and inhibit bladder emptying.

The anticholinergic effects of the medication are impairing normal bladder function. Many over-the-counter cold medications have an anticholinergic effect that interferes with normal bladder emptying. These effects on micturition are not a result of cholinergic actions or miscommunication between the pontine micturition center and the spinal cord. Stimulation of the parasympathetic nervous system would tend to increase rather than decrease bladder emptying.

A nurse is caring for a client who has just experienced a spinal cord injury. Which of the following bladder dysfunctions will the client experience? a) The bladder become hyperactive. b) Internal sphincter contracts. c) External sphincter relaxes. d) The bladder becomes atonic.

The bladder becomes atonic. The immediate and early effects of spinal cord injury on bladder function are quite different from those that follow recovery from the initial injury. During the period immediately after spinal cord injury, a state of spinal shock develops in which all reflexes, including the micturition reflex, are depressed. During this stage, the bladder becomes atonic and cannot contract. Catheterization is necessary to prevent injury to urinary structures associated with overdistention of the bladder. Depression of reflexes lasts from a few weeks to 6 months (usually 2 to 3 months), after which the spinal reflexes return and become hyperactive

The nurse is caring for a client who has produced an average of 20 ml/hour for the previous day. The nurse recognizes this compares in which way to the normal urine output? a) The kidneys should produce about 1.5 liters of urine each day. b) The normal kidney produces an average 3000 ml of urine daily. c) The kidneys should produce a minimum of 10 ml/hour over one day. d) This represents normal urinary output for 24 hours.

The kidneys should produce about 1.5 liters of urine each day. Explanation: The kidneys normally produce approximately 1.5 L or 1500 ml of urine each day.

Which of the following patients should have a feeling of bladder fullness? a) The patient with 200 mL of urine in their bladder b) The patient who has neuropathy c) The patient who is incontinent d) The patient with 25 mL of urine in their bladder

The patient with 200 mL of urine in their bladder Explanation: In a normally functioning bladder, the sensation of bladder fullness is first perceived when the bladder contains 100-200 mL of urine. The patient who has neuropathy or is incontinent may not have a feeling of bladder fullness.

A nurse observes that a patient's urine is cola colored and considers which of the following as a possible reason? a) The patient has ingested a dark-colored drink. b) The patient's urine contains material from the degradation of red blood cells. c) The patient has an elevation of urine potassium. d) The patient's urine has a decrease in the specific gravity.

The patient's urine contains material from the degradation of red blood cells. When red blood cells degrade in the urine, urine may appear cola colored.

A young woman presents with signs and symptoms of urinary tract infection (UTI). The nurse notes that this is the fifth UTI in as many months. What would this information lead the nurse to believe? a) The woman takes too many bubble baths. b) The woman has multiple sexual partners. c) There is possible obstruction in the urinary tract. d) The woman does not clean herself properly.

There is possible obstruction in the urinary tract. Urinary tract obstruction encourages the growth of microorganisms and should be suspected in persons with recurrent UTIs. The other answers can cause lower UTIs, but an obstruction would be considered because of the frequency of the infections.

The nurse is teaching a group of nursing students about the mechanism of action of common diuretics. Which of these best reflects the mechanism of these drugs? a) They block uric acid and glucose in the renal tubules. b) They block the reabsorption of sodium and chloride in the nephron. c) They enhance absorption of potassium in the loop of Henle. d) They promote release of aldosterone from the adrenal glands.

They block the reabsorption of sodium and chloride in the nephron. Explanation: Most diuretics share the same mechanism of action—blockade of sodium and chloride reabsorption. By blocking the reabsorption of these solutes, diuretics create an osmotic pressure gradient within the nephron that prevents the passive reabsorption of water. Thus, diuretics cause water and sodium to be retained in the nephron, promoting the excretion of both.

Which of the following medications may be responsible for a client developing increased uric acid levels by decreasing ECF volume? a) Penicillin antibiotics b) Thiazide diuretics c) Maalox products d) Vitamin C

Thiazide diuretics Explanation: Because of its effect on uric acid secretion, aspirin is not recommended for treatment of gouty arthritis. Thiazide and loop diuretics also can cause hyperuricemia and gouty arthritis, presumably through a decrease in ECF volume and enhanced uric acid reabsorption.

Bruising of the skin is a manifestation of bleeding disorders in patients with CKD. a) True b) False

True Bleeding disorders manifested by persons with CKD include epistaxis, menorrhagia, gastrointestinal bleeding, and bruising of skin and subcutaneous tissues.

The nursing student, while studying anatomy and physiology, correctly identifies which of the following to be responsible for carrying urine to the bladder? a) Ureters b) Bowman's capsule c) External sphincter d) Trigone

Ureters Correct Explanation: Urine passes from the kidneys to the bladder through the ureters. The bowman's capsule is a component of the kidney. The external sphincter serves as a reserve mechanism to stop micturition when it is occurring, while the trigone is the smooth triangular area that is bounded by the openings of the ureters and the urethra.

The nurse understands that medications although very beneficial to clients can also have harmful effects. When working with elderly clients the nurse should recognize which of the following is a common result of potent, fast-acting diuretics? a) Decreased urine output b) Increased potassium levels c) No untoward effects d) Urge incontinence

Urge incontinence Medications prescribed for other health problems may prevent a healthy bladder from functioning properly. Potent, fast-acting diuretics are known for their ability to cause urge incontinence. They can decrease potassium levels and increase urine output. They do have untoward effects

Gout and the development of kidney stones are often attributed to high levels of what compound? a) Albumin b) Uric acid c) Urea d) Protein

Uric acid Explanation: High blood levels of uric acid can cause gout and excessive urine levels can cause kidney stones. High albumin, urea, and protein levels can be indicative of renal disease, but are not specifically related to gout and kidney stones the way uric acid is

A patient is admitted with lower urinary tract obstruction and stasis. Which of the following is the primary intervention? a) Laxative administration b) Administration of intravenous fluids c) Urinary catheterization d) Increased oral fluids

Urinary catheterization Explanation: The relief of lower urinary tract obstruction is directed toward relief of bladder distension through urinary catheterization. This is the primary intervention. The other interventions are not the priority.

When teaching a community education class about the 7 warning signs of cancer, the nurse will note that the most common sign of bladder cancer is: a) Inability to empty the bladder fully. b) Painless bloody urine. c) Passage of large clots after voiding. d) Colic spasms of the ureters.

Urinary catheters are a source of urethral irritation and provide a means for entry of microorganisms into the urinary tract. Catheter-associated bacteriuria remains the most frequent cause of gram-negative septicemia in hospitalized clients. Studies have shown that bacteria adhere to the surface of the catheter and initiate the growth of a biofilm that then covers the surface of the catheter.

The nursing instructor, while teaching about incontinence in older adults, informs the students that which of the following is true? a) Urinary incontinence is a problem found only in the older adult population. b) Urinary incontinence is a common problem in older adults, both male and female. c) Urinary incontinence is found only in males as they age. d) Urinary incontinence is found only in females as they age.

Urinary incontinence is a common problem in older adults, both male and female. Urinary incontinence is a common problem in older adults both male and female. It can also occur at an earlier age in both sexes.

Which of the following substances is most likely to be reabsorbed in the tubular segments of the nephron using passive transport mechanisms? a) Water b) Sodium c) Phosphate d) Calcium

Water Explanation: Water is passively reabsorbed across tubular epithelial membranes while ions like sodium, phosphate and calcium necessitate active transport.

A female client asks, "Why do I leak urine every time I cough or sneeze?" The health care worker's response is based on which physiologic principle? a) When intravesical pressure exceeds maximal urethral closure pressure b) Involuntary bladder continence during filling c) A decrease in bladder distensibility d) A pressure difference between the urethra and bladder

When intravesical pressure exceeds maximal urethral closure pressure Correct Explanation: Stress incontinence represents the involuntary loss of urine that occurs when, in the absence of detrusor muscle action, the intravesical pressure exceeds the maximum urethral closure pressure. Stress incontinence, which is a common problem in women of all ages, occurs as the result of weakness or disruption of pelvic floor muscles, leading to poor support of the vesicourethral sphincters. Except during the act of micturition, intraurethral pressure is normally greater than intravesical pressure. Urge incontinence and overactive bladder are associated with urgency caused by bladder infection or CNS or myogenic mechanisms. Overflow incontinence is an involuntary loss of urine that occurs when intravesical pressure exceeds the maximal urethral pressure because of bladder distention in the absence of detrusor activity

Prior to undergoing diagnostic testing with contrast, it is recommended that older adult clients have their creatinine level checked. The rationale for this is to ensure the client: a) Is not allergic to shell fish or iodine b) Does not have a kidney stone obstructing the urethra c) Is in good enough health to withstand a walking on a treadmill d) Will not undergo an acute kidney injury by decreasing renal blood flow

Will not undergo an acute kidney injury by decreasing renal blood flow Some drugs, such as diuretics, high molecular weight radiocontrast media, the immunosuppressive drugs cyclosporine and tacrolimus, and the nonsteroidal anti-inflammatory drugs (NSAIDs), can cause acute kidney injury by decreasing renal blood flow. Checking creatinine levels do not predict the client's allergies, a kidney stone, or tolerance for stress testing.

A female teenager has experienced three uncomplicated urinary tract infections in the last 3 months. Knowing the anatomical location of the urethra, the nurse should educate this teenager about: a) Wearing gloves when wiping perineum after defecation to prevent Staphylococcus aureus infection b) Proper handwashing to decrease amount of Pseudomonas growing on the hands c) Wiping from front to back to preventEscherichia coli contamination of the urethra d) Washing hands prior to inserting a tampon to minimize the risk of group B Streptococcus

Wiping from front to back to preventEscherichia coli contamination of the urethra Correct Explanation: Most commonly, urinary tract infections (UTIs) are caused by Escherichia coli that enter through the urethra. Escherichia coli are abundant in fecal matter. Other uropathic pathogens include Staphylococcus saprophyticus in uncomplicated UTIs and both non-E. coli gram-negative rods (Proteus mirabilis, Klebsiella pneumoniae, Pseudomonas) and gram-positive cocci (Staphylococcus aureus, group B Streptococcus) in complicated UTIs.

The client with chronic kidney disease asks the nurse why he must take active vitamin D (calcitriol) as a medication. Which of these is the most appropriate response by the nurse? a) With renal disease vitamin D is unable to be transformed to its active form. b) The skin is no longer able to activate vitamin D for use. c) Vitamin D can no longer be released from the exocrine glands. d) The bones no longer respond to vitamin D and demineralize.

With renal disease vitamin D is unable to be transformed to its active form. Explanation: Cholecalciferol and ergocalciferol must undergo chemical transformation to become active: first to 25-hydroxycholecalciferol in the liver and then to 1,25-dihydroxycholecalciferol in the kidneys. Individuals with end-stage renal disease are unable to transform vitamin D to its active form and may require pharmacologic preparations of the active vitamin (calcitriol) for maintaining mineralization of their bones.

A child has been brought to an urgent care clinic. The parents state that the child is "not making water." When taking a history, the nurse learns the child had a sore throat about 1 week ago but seems to have gotten over it. "We [parents] only had to give antibiotics for 3 days for the throat to be better." The nurse should suspect the child has developed: a) acute renal failure. b) acute postinfectious glomerulonephritis. c) nephrotic syndrome. d) kidney stones.

acute postinfectious glomerulonephritis. The classic case of poststreptococcal glomerulonephritis follows a streptococcal infection by approximately 7 to 12 days: the time needed for the development of antibodies. The primary infection usually involves the pharynx (pharyngitis), but can also result from a skin infection (impetigo). Oliguria, which develops as the GFR (Glomerular filtration rate ) decreases, is one of the first symptoms.

In the emergency department, a client arrives following a car accident. His pulse is 122; BP 88/60; respiration is 18 bpm. Urine output is 4 mL over the first hour on arrival. When in shock, this lower urine output is primarily due to: a) innervation of the sympathetic nervous system causing constriction of the afferent arteriole. b) obstruction in the glomerular capillaries due to overabundance of large molecules being released. c) parasympathetic stimulation causing the renal arteries to constrict in response to pain. d) high filtration rates in the glomerulus.

innervation of the sympathetic nervous system causing constriction of the afferent arteriole. Explanation: During periods of strong sympathetic stimulation, such as shock, constriction of the afferent arteriole causes a marked decrease in renal blood flow, and thus glomerular filtration pressure. Consequently, urine output can fall almost to zero. The location of the glomerulus between two arterioles allows for maintenance of a high-pressure filtration system. The glomerular filtrate has a chemical composition similar to plasma, but contains almost no proteins because large molecules do not readily pass through the openings in the glomerular capillary wall.

A client with chronic kidney disease has developed cardiac calcification. On admission the priority assessment would be for the nurse to: a) place on a heart monitor to watch for arrhythmias. b) assess for pulses in the feet bilaterally. c) listen to the heart for extra clicking sounds. d) check the BP in both arms and compare.

place on a heart monitor to watch for arrhythmias. Visceral calcification may be found in the myocardium, lungs, and stomach. In cardiac calcification, the deposits usually develop in the conduction system and may result in serious cardiac arrhythmias. -The other assessments are valid, however watching for arrhythmias is the priority.

A client who is having problems with his bladder and kidneys is scheduled for a cystoscopy. He asks the nurse why he has to have the test. Which of the following would be the best response by the nurse? a) "A cystoscopy provides information about the flow rate during urination." b) "A cystoscopy can estimate bladder volume." c) "A cystoscopy enables direct visualization of the urethra, bladder and ureteral orifices." d) "A cystoscopy provides information about bladder emptying."

• "A cystoscopy enables direct visualization of the urethra, bladder and ureteral orifices." A cystoscopy enables direct visualization of the urethra, bladder and ureteral orifices. A bladder scan provides method for estimating volume and a PVR provides information bout bladder emptying. Flow rate is determined by doing uroflowmetry

A patient with a recent diagnosis of renal failure that will require hemodialysis is being educated by the nurse in the dietary management of the disease. Which of the patient's following statements shows an accurate understanding of this component of treatment? Select all that apply. a) "I'm going to try a high-protein, low-carbohydrate diet." b) "I've made a list of high-phosphate foods so that I can try to avoid them." c) "I don't think I've been drinking enough, so I want to include 8 to 10 glasses of water each day." d) "I'll increase the carbohydrates in my diet to provide sufficient energy." e) "I'm making a point of trying to eat lots of bananas and other food rich in potassium."

• "I'll increase the carbohydrates in my diet to provide sufficient energy." • "I've made a list of high-phosphate foods so that I can try to avoid them." Persons with chronic kidney disease (CKD) are usually encouraged to limit their dietary phosphorus as a means of preventing secondary hyperparathyroidism, renal osteodystrophy, and hypocalcemia. Excessive protein, potassium, and fluids can be detrimental in individuals whose kidney disease requires hemodialysis. Because protein intake is limited, carbohydrate consumption should increase to meet daily energy requirements.

Which of the following patients scheduled for an interventional radiology procedure requiring administration of radiocontrast dye would be considered high risk for nephrotoxicity? Select all that apply. a) 25 year old with a history of glomerular nephritis who is complaining of severe flank pain. b) 14 year old with severe abdominal pain. c) 67 year old diabetic undergoing diagnostic testing for new-onset proteinuria. d) 45 year old with elevated liver enzymes possibly due to fatty liver cirrhosis. e) 53 year old male undergoing biopsy for a suspicious "spot" on their chest x-ray.

• 25 year old with a history of glomerular nephritis who is complaining of severe flank pain. • 67 year old diabetic undergoing diagnostic testing for new-onset proteinuria. Radiocontrast media-induced nephrotoxicity is thought to result from direct tubular toxicity and renal ischemia. The risk for renal damage caused by radiocontrast media is greatest in older adults and those with pre-existing kidney disease, volume depletion, diabetes mellitus, and recent exposure to other nephrotoxic agents

Which of the following individuals are displaying identified risk factors for the development of lower urinary tract obstruction? Select all that apply. a) 68 year-old man who has been diagnosed with benign prostatic hyperplasia (BPH). b) 20 year-old man who has spina bifida and consequent impaired mobility. c) 74 year-old woman who has developed a lower bowel obstruction following several weeks of chronic constipation. d) 55 year-old man with diabetes who is receiving diuretic medications for the treatment of hypertension. e) 32 year-old woman who had a healthy delivery of her third child 4 months ago. f) 30 year-old woman who has been diagnosed with gonorrhea.

• 68 year-old man who has been diagnosed with benign prostatic hyperplasia (BPH). • 30 year-old woman who has been diagnosed with gonorrhea. • 74 year-old woman who has developed a lower bowel obstruction following several weeks of chronic constipation. • 20 year-old man who has spina bifida and consequent impaired mobility. Explanation: BPH frequently obstructs the urethra, while sexually transmitted diseases, bowel obstructions and spina bifida are also associated with physical blockages of the lower urinary tract. Postpartum women and individuals receiving diuretics would be more likely to be at risk for incontinence rather than urinary retention.

The nurse knows that a patient with chronic kidney disease (CKD) may develop renal osteodystrophy (skeletal damage). Which of the following are characteristic of both high-bone-turnover and low-bone-turnover osteodystrophy? Select all that apply. a) Decreased osteoblast and osteoclast proliferation b) Bone marrow fibrosis c) Defective bone remodeling d) Abnormal bone resorption e) Increased osteoblast and osteoclast proliferation

• Abnormal bone resorption • Defective bone remodeling Abnormal bone resorption and defective bone remodeling are characteristic of both high-bone-turnover and low-bone-turnover osteodystrophy. In high-turnover, there is increased osteoblast and osteoclast proliferation while in low-turnover there is decreased proliferation. Bone marrow fibrosis is characteristic of high-bone turnover.

If a client with a kidney stone has the "classic" ureteral colic, the client will describe his pain as: Select all that apply. a) Excruciating b) In the flank and upper outer quadrant of the abdomen c) Diffuse over the entire lower back and legs d) Acute, intermittent

• Acute, intermittent • Excruciating • In the flank and upper outer quadrant of the abdomen The symptoms of renal colic are caused by stones 1 to 5 mm in diameter that can move into the ureter and obstruct flow. Classic ureteral colic is manifested by acute, intermittent, and excruciating pain in the flank and upper outer quadrant of the abdomen on the affected side. The pain may radiate to the lower abdominal quadrant, bladder area, perineum, or scrotum in the man. The pain is usually not described as diffuse and over the entire low back and legs.

Sexual dysfunction in people with CKD is thought to be multifactorial. What are thought to be causes of sexual dysfunction in people with CKD? (Select all that apply.) a) Uremic toxins b) Antihypertensive drugs c) Psychological factors d) Inability to vasodilate veins e) High incidence of sexually transmitted diseases

• Antihypertensive drugs • Psychological factors • Uremic toxins The cause of sexual dysfunction in men and women with CKD is unclear. The cause probably is multifactorial and may result from high levels of uremic toxins, neuropathy, altered endocrine function, psychological factors, and medications (e.g., antihypertensive drugs). The other answers do not apply in this situation.

An elderly woman comes to the hospital and is diagnosed with urinary obstruction and retention. Which of the following symptoms would the nurse expect this client to demonstrate? Select all that apply. a) Strong stream b) Frequency c) Overflow incontinence d) Hesitancy e) Nladder distention

• Bladder distention • Hesitancy • Frequency • Overflow incontinence Signs of outflow obstruction and urine retention are: bladder distention, hesitancy, straining when initiating urination, small and weak stream, frequency, feeling of incomplete bladder emptying and overflow incontinence. A strong stream is not a symptom.

An elderly client has just been admitted for urinary tract obstruction and retention. Which of the following are symptoms that the nurse should suspect this client to demonstrate? Select all that apply. a) Hesitancy b) Bladder distention c) Small stream d) Forceful stream e) Overflow incontinence

• Bladder distention • Hesitancy • Small stream • Overflow incontinence Signs of outflow obstruction and urine retention are bladder distention, hesitancy, straining when initiating urination, small and weak stream, frequency, feeling of incomplete bladder emptying, and overflow incontinence

A patient has developed chronic kidney disease (CKD). The nurse will advise the patient to be alert for the development of which hematologic signs and symptoms of this disorder? Select all that apply. a) Fatigue b) Bradycardia c) Gastrointestinal bleeding d) Bruising e) Intravascular clotting

• Bruising • Gastrointestinal bleeding • Fatigue Explanation: In CKD, platelet function is impaired and bruising and gastrointestinal bleeding can occur. Intravascular clotting usually does not happen. Decreased renal production of erythropoietin results in decreased synthesis of red blood cells, resulting in anemia and fatigue. Decrease in red cells also decreases blood viscosity and decreases cerebral oxygen delivery, both of which contribute to tachycardia rather than bradycardia.

An adult client presents to the emergency department with manifestations of acute cystitis. For which would the nurse assess the client? Select all that apply. a) Urinary infrequency b) Burning on urination c) Chest pain d) Fever e) Cloudy urine f) Back discomfort

• Burning on urination • Cloudy urine • Back discomfort An acute episode of cystitis (bladder infection) is characterized by frequency of urination, lower abdominal or back discomfort, and burning and pain on urination (i.e., dysuria). Occasionally, the urine is cloudy and foul smelling. In adults, fever and other signs of infection usually are absent. If there are no complications, the symptoms disappear within 48 hours of treatment.

A client is diagnosed with decreased gomerular filtration rate but has no renal damage. The nurse recognizes that this can occur with which of the following? Select all that apply. a) Carbohydrate diet b) Heart failure c) Dehydration d) Cirrhosis e) Removal of one kidney

• Cirrhosis • Heart failure • Removal of one kidney • Dehydration Chronic reduction in glomerular filtration rate with no renal damage can be the result of depleted body fluid, removal of one kidney, and conditions such as heart failure and cirrhosis which decrease renal perfusion. A carbohydrate diet should not affect GFR.

If a CKD client is developing uremic encephalopathy, the earliest manifestations may include: Select all that apply. a) Delirium and hallucinations b) New-onset seizures c) Diminished awareness d) Decreased alertness

• Decreased alertness • Diminished awareness Reductions in alertness and awareness are the earliest and most significant indications of uremic encephalopathy. - Late in the disease process, the client may develop delirium, coma, and seizures.

Anemia frequently occurs in clients with chronic kidney disease (CKD). The nurse will monitor these clients for which of the following contributors to anemia? Select all that apply. a) Decreased erythropoietin production b) Chronic blood loss c) Iron overload d) Anorexia e) Bone marrow suppression

• Decreased erythropoietin production • Anorexia • Bone marrow suppression • Chronic blood loss Production of erythropoietin, which stimulates red cell synthesis, becomes deficient as the kidneys fail. Iron deficiency can develop as anorexia decreases ingestion of foods containing this mineral. Bone marrow production of red cells is suppressed as uremia develops. Clients on dialysis will experience chronic blood loss

A patient is diagnosed with chronic kidney disease (CKD). The nurse recognizes that this patient will experience which of the following? Select all that apply. a) Decreased tubular reabsorption b) Hypophospatemia c) Decreased glomerular filtration d) Proliferation of nephrons e) Decreased renal endocrine function

• Decreased renal endocrine function • Decreased tubular reabsorption • Decreased glomerular filtration Chronic kidney disease results in loss of nephrons, with a decrease in tubular reabsorption, glomerular filtration, and endocrine function. -Phosphate accumulates in the blood, as the kidneys lose their ability to excrete this electrolyte.

The nurse is preparing to assess a client who has just been admitted to the hospital with a diagnosis of prerenal failure. Which would the nurse expect the client to manifest? Select all that apply. a) BUN-to-serum creatinine ratio of greater than 20:1 b) Increased urinary output c) Decreased urinary output d) Increased BUN e) Decreased BUN f) BUN to serum creatinine ratio of 10:1

• Decreased urinary output • Increased BUN • BUN-to-serum creatinine ratio of greater than 20:1 Prerenal injury is manifested by a sharp decrease in urine output and a disproportionate elevation of blood urea nitrogen (BUN) in relation to serum creatinine levels. Consequently, there also is a disproportionate elevation in the ratio of BUN to serum creatinine, from a normal value of 10:1 to a ratio greater than 20:1.

A client who is experiencing bladder overfilling will more than likely have which of the following disease? Select all that apply. a) Diabetic neuropathies b) Multiple sclerosis c) Parkinson's disease d) Spinal cord injury e) Stroke

• Diabetic neuropathies • Multiple sclerosis Explanation: Common causes of bladder overfilling that lead to a loss of ability to perceive bladder filling are diabetic neuropathies and multiple sclerosis. Spinal cord injury results in a loss of external sphincter tone or areflexic bladder where it fills but not contract. Parkinson's causes detrusor contractions suddenly without warning and are difficult to control while a stroke causes a loss of ability to perceive bladder filling.

When conducting an admission interview with a client with a history of urinary incontinence, the nurse will specifically ask whether the client is prescribed which classification of medications in order to determine a possible cause? Select all that apply. a) Tricyclic antidepressants b) Sedatives c) Diuretics d) Hypnotics e) Acetylcholines

• Diuretics • Hypnotics • Sedatives Medication prescribed for other health problems may prevent a healthy bladder from functioning normally. Drugs such as hypnotics, tranquilizers, and sedatives can interfere with the conscious inhibition of voiding, leading to urge incontinence. Diuretics, particularly in older adults, increase the flow of urine and may contribute to incontinence, particularly in people with diminished bladder capacity and in those who have difficulty reaching the toilet. Acetylcholines and tricyclic antidepressants are sometimes used to treat specific types of incontinence.

A child is diagnosed with chronic kidney disease (CKD). The nurse knows that which of the following statements regarding supportive renal therapy in children are true? Select all that apply.

• Early transplantation is the preferred treatment when considering long-term effects. • Children older than 12 years will benefit from hemodialysis. • Peritoneal dialysis is generally most appropriate for children younger than 6. Children between birth and 5 years of age often are managed with peritoneal dialysis. -For those older than 12 years of age hemodialysis is usually preferred. -Early transplantation in young children is the best choice to promote physical growth, improve cognitive function, and foster psychosocial development. -Immunosuppressive therapy in children is similar to that required in adults. -Corticosteroids such as prednisone have risk for growth retardation.

A client with chronic kidney disease (CKD) is anemic. The nurse will attempt to alleviate the anemia in order to prevent which of the following? Select all that apply. a) Decreased myocardial oxygen b) Increased blood viscosity c) Hypersomnia d) Tachycardia e) Fatigue

• Tachycardia • Fatigue • Decreased myocardial oxygen Uncorrected anemia provokes fatigue and insomnia, a decrease in blood viscosity, a decrease in myocardial oxygen supply, and tachycardia as the heart attempts to supply sufficient oxygen to the heart and brain

Which client would be considered at a higher risk for developing Goodpasture Syndrome, an aggressive form of glomerulonephritis? Select all that apply. a) Elderly person with a recent influenza infection b) Painter working with a busy construction crew who finishes all the house painting indoor and out c) Electrician who wires homes and businesses for high-definition Internet d) Hair style expert who specializes in coloring [dyes] hair e) Middle-aged over-the-road truck driver

• Elderly person with a recent influenza infection • Painter working with a busy construction crew who finishes all the house painting indoor and out • Hair style expert who specializes in coloring [dyes] hair The cause of the disorder [Goodpasture Syndrome] is unknown, although influenza infection and exposure to hydrocarbon solvent (found in paints and dyes) have been implicated in some persons, as have various drugs and cancers. There is not a greater risk associated with truck drivers or electricians.

The nurse is caring for a patient who has not received prenatal care during her pregnancy. The patient delivers a newborn who dies an hour after birth because of total agenesis of both kidneys. Which of the following physical features does the newborn exhibit that correlates with this diagnosis? Select all that apply. a) Imperforate anus b) Broad and flat nose c) Low-set ears d) Cleft lip e) Eyes widely separated with epicanthic folds

• Eyes widely separated with epicanthic folds • Low-set ears • Broad and flat nose Total agenesis of both kidneys is incompatible with extrauterine life. Infants are stillborn or die shortly after birth of pulmonary hypoplasia. Newborns with renal agenesis often have characteristic facial features, sometimes called Potter syndrome, resulting from the effects of oligohydramnios. The eyes are widely separated and have epicanthic folds, the ears are low set, the nose is broad and flat, the chin is receding, and limb defects are often present.

The nurse knows that uremia or "urine in the blood" is often used to describe the clinical manifestations of chronic kidney disease (CKD). Which of the following are early signs and symptoms of uremia? Select all that apply. a) Elevated blood glucose b) Nausea c) Apathy d) Fatigue e) Weakness

• Fatigue • Nausea • Apathy • Weakness As nitrogenous wastes accumulate in the circulatory system, symptoms of fatigue, nausea, apathy, and weakness will appear. CKD does not immediately affect blood glucose.

Glomerular filtration rate (GFR) is the best indicator of renal function. The nurse knows that GFR can vary with which of the following factors? Select all that apply. a) Age b) Body size c) Diet d) Ethnicity e) Gender

• Gender • Age • Body size • Ethnicity GFR, the best indicator of overall renal function, varies with age, gender, body size, and ethnicity. Equations are available for calculating GFR based on serum creatinine and these variables. Diet, if it includes sufficient fluid intake to maintain hydration, should not affect GFR.

A client has been recently undergone diagnostic testing for possible Berger disease. The nurse caring for this client would anticipate the primary clinical manifestations include which of the following? Select all that apply. a) Fever, chills, and general body aches b) Gross hematuria c) Recent upper respiratory infection d) Elevated ketone levels in the urine

• Gross hematuria • Recent upper respiratory infection • Fever, chills, and general body aches Early in the disease, many people with the disorder have no obvious symptoms, and the disorder is discovered during screening or examination for another condition. In others, the disorder presents with gross hematuria that is preceded by upper respiratory tract infection, GI tract symptoms, or flulike illness. The hematuria lasts 2 to 6 days. Elevated ketones are usually associated with acidosis, fasting, high-protein diet, or diabetes to name a few.

A child has received a kidney transplant at the age of 3. Knowing he will be on immunosuppressive agents like corticosteroids, the nurse should educate the parents about which long-term side effects? Select all that apply. a) Growth retardation b) Excess weight loss c) Development of cataracts d) Hypertension e) Frequent hematuria

• Hypertension • Growth retardation • Development of cataracts Early transplantation in young children is regarded as the best way to promote physical growth, improve cognitive function, and foster psychosocial development. Immunosuppressive therapy in children is similar to that used in adults. All immunosuppressive agents have side effects, including increased risk for infection. Corticosteroids carry the risk for hypertension, orthopedic complications (especially aseptic necrosis), cataracts, and growth retardation. Steroids usually are associated with weight gain. -Frequent hematuria is caused by something other than steroids and should be reported.

Which of the following have the potential to cause chronic kidney disease? (Select all that apply.)

• Hypertension • Diabetes • Glomerulonephritis CKD can result from a number of conditions including diabetes, hypertension, glomerulonephritis as well as systemic lupus erythematosus, and polycystic kidney disease. These conditions slowly but steadily destroy renal tissue resulting in irreversible loss of function. Cardiomyopathies do not cause renal failure; they cause heart failure.

Chronic kidney disease (CKD) can cause derangement of calcium and phosphate metabolism. The nurse will monitor a client with CKD for which of the following? Select all that apply. a) Decreased blood levels of phosphate b) Decreased blood levels of calcium c) Excess calcium deposits in bone d) Decreased release of parathyroid hormone (PTH) e) Impaired phosphate excretion

• Impaired phosphate excretion • Decreased blood levels of calcium In CKD, renal ability to excrete phosphate declines. As blood levels of phosphate increase, levels of calcium inversely related to phosphate, fall. This provokes an increase in PTH, which stimulates mobilization of calcium from bone.

The nursing students have learned in class that causes of urinary obstruction and urinary incontinence include which of the following? Select all that apply. a) Structural changes in the pancreas b) Impairment of neurologic control of bladder function c) Structural changes of the gallbladder d) Structural changes in the urethra e) Structural changes in the bladder

• Impairment of neurologic control of bladder function • Structural changes in the urethra • Structural changes in the bladder Correct Explanation: Urinary obstruction and urinary incontinence can be caused by several factors, including structural changes in the bladder, structural changes in the urethra, and impairment of neurologic control of bladder function. Changes in the gallbladder or pancreas do not cause urinary obstruction or incontinence.

A client is diagnosed with acute renal injury. The nurse will evaluate the client for which of the following possible causes for this disorder? Select all that apply. a) Hormonal imbalance b) Ischemic injury c) Obstruction of urinary outflow d) Nephrotoxic substances e) Nonischemic reduced renal blood flow

• Ischemic injury • Nonischemic reduced renal blood flow • Obstruction of urinary outflow • Nephrotoxic substances Acute renal injury can result from ischemia, reduced renal blood without ischemia, nephrotoxic substances including some drugs, and obstruction of urinary outflow. Hormonal imbalance usually does not cause renal injury.

A client is diagnosed with renal failure. The nurse must monitor for failure of which of the following functions? Select all that apply. a) Electrolyte balance b) Removal of metabolic waste from blood c) Adrenal secretion d) Acid-base balance e) Maintenance of body water

• Maintenance of body water • Electrolyte balance • Acid-base balance • Removal of metabolic waste from blood Functions of the kidney include removal of metabolic end products and regulation of body water, electrolyte balance and acid-base balance. -Although the adrenal glands are located above the kidneys, the kidneys are not directly involved in their regulation.

A chronic kidney disease client who has renal osteodystrophy should be assessed for which of the following complications? Select all that apply. a) Stress fractures b) Bone pain c) Muscle weakness d) Urosepsis e) Kidney stones

• Muscle weakness • Bone pain • Stress fractures Both types of renal osteodystrophy are manifested by abnormal absorption and defective bone remodeling. Renal osteodystrophy is typically accompanied by reductions in bone mass, alterations in bone microstructure, bone pain, and skeletal fracture. There are changes in bone turnover, mineralization, and bone volume, accompanied by bone pain and muscle weakness, risk of fractures, and other skeletal complications. -Kidney stones and urosepsis are not associated with renal osteodystrophy.

A client is diagnosed with chronic kidney disease (CKD). The nurse recognizes that which of the following statements regarding CKD are correct? Select all that apply. a) Functioning nephrons compensate for those that are damaged. b) The rate of nephron destruction is the same in all clients. c) Signs and symptoms develop gradually. d) There is a reduction in glomerular filtration rate (GFR) as nephrons are destroyed. e) Nephron destruction takes place over many months.

• Nephron destruction takes place over many months. • Signs and symptoms develop gradually. • Functioning nephrons compensate for those that are damaged. • There is a reduction in glomerular filtration rate (GFR) as nephrons are destroyed. In CKD, the rate of nephron destruction occurs over many months and is different among clients. Signs and symptoms develop gradually as nephrons are destroyed and functioning nephrons gradually lose the ability to compensate for this. As nephrons are destroyed, GFR declines. (

As nitrogenous wastes increase in the blood, the CKD client may exhibit which of the following clinical manifestations? Select all that apply. a) Pruritis b) Extremely low platelet counts c) Photophobia d) Numbness in lower extremities e) Restless leg syndrome

• Numbness in lower extremities • Restless leg syndrome • Pruritis The uremic state is characterized by signs and symptoms of altered neuromuscular function (e.g., fatigue, peripheral neuropathy, restless leg syndrome, sleep disturbances, uremic encephalopathy); gastrointestinal disturbances such as anorexia and nausea; white blood cell and immune dysfunction, and dermatologic manifestations such as pruritus. Photophobia and thrombocytopenia are usually not associated with CKD.

The nurse knows that a patient with chronic kidney disease (CKD) may experience which of the following changes in skin integrity? Select all that apply. a) Pale skin b) Decreased perspiration c) Brittle fingernails d) Moist skin and mucous membranes e) Increased oil gland secretion

• Pale skin • Brittle fingernails • Decreased perspiration In CKD, anemia due to loss of erythropoietin activity causes pale skin. Perspiration and oil secretion are decreased, leading to dry skin. Fingernails become brittle.

Which of the following patients does the nurse need to monitor for the development of intense intrarenal vasoconstriction that may induce prerenal failure? Select all that apply. a) Patient taking cyclosporine to prevent rejection for a liver transplant b) Patient who is HIV positive c) Patient undergoing cardiac catheterization d) Patient with an elevated blood urea nitrogen who is taking ibuprofen every 6 hours for back pain e) Patient taking acetaminophen for fever

• Patient undergoing cardiac catheterization • Patient taking cyclosporine to prevent rejection for a liver transplant • Patient with an elevated blood urea nitrogen who is taking ibuprofen every 6 hours for back pain Some vasoactive mediators, drugs, and diagnostic agents stimulate intense intrarenal vasoconstriction and can induce glomerular hypoperfusion and prerenal failure. Examples include endotoxins, radiocontrast agents such as those used for cardiac catheterization, cyclosporine, and nonsteroidal anti-inflammatory drugs. -NSAIDs can reduce renal blood flow by inhibiting prostaglandin syntheses. In some persons with diminished renal perfusion, NSAIDs can precipitate prerenal failure. -Acetaminophen for short-term use does not predispose a patient to prerenal failure. -An HIV+ client is at no more risk than any other patient.

Which manifestations are most common in clients with COPD that is predominantly chronic bronchitis? Select all that apply. a) Cyanosis b) Increased A-P diameter c) Increased mucus secretion d) Hyperresonance e) Peripheral edema

• Peripheral edema • Increased mucus secretion • Cyanosis Explanation: In chronic bronchitis, there is an increase in mucus production, cyanosis, and often fluid retention associated with right heart failure. Emphysema is noted for its loss of lung elasticity and enlargement of the distal air spaces. This LEADS TO hyperresonance, decreased breath sounds, and an increase in the A-P chest diameter, commonly called barrel chest. Clients with emphysema tend to take short frequent breaths and exhale through pursed lips.

A client is diagnosed with chronic kidney disease (CKD). The nurse will monitor this client for which of the following Select all that apply. a) Hyponatremia b) Hypocalcemia c) Polyuria d) Hyperkalemia e) Metabolic alkalosis

• Polyuria • Hyperkalemia • Hypocalcemia • Hyponatremia The failing kidneys lose ability to concentrate urine and to reabsorb sodium. Hyperkalemia develops late in CKD, as nephrons can no longer regulate potassium excretion. Metabolic acidosis occurs when balance between sodium and bicarbonate is lost. Hypocalcemia develops as excretion of phosphate fails and blood levels of phosphate rise.

A client has a tubulointerstitial disorder and is no able to concentrate urine. The nurse will likely assess which manifestations associated with this disorder? Select all that apply. a) Acidic urine b) Nocturia c) Hematuria d) Polyuria

• Polyuria • Nocturia The tubulointerstitial disorders are distinguished clinically from glomerular diseases by the absence, in the early stages, of such hallmarks of nephritis and nephrosis as hematuria and proteinuria, and by the presence of disorders in tubular function. These disorders, which are often subtle, include the inability to concentrate urine, as evidenced by polyuria and nocturia; interference with acidification of urine, resulting in metabolic acidosis; and diminished tubular reabsorption of sodium and other substances.

Congenital disorders of the kidneys are fairly common, occurring in approximately 1:1000 live births. What is the result to the newborn when bilateral renal dysplasia occurs? (Select all that apply.) a) Oligohydramnios b) Potter facies c) Pulmonary hypoplasia d) Multicystic kidneys e) Renal failure

• Potter facies • Oligohydramnios • Pulmonary hypoplasia • Renal failure Bilateral renal dysplasia causes oligohydramnios and the resultant Potter facies, pulmonary hypoplasia, and renal failure. Multicystic kidneys are a disorder, not the result of a congenital problem.

A nurse is caring for a child with Wilms tumor, stage I. Which of the following can be said regarding this diagnosis? Select all that apply. a) The tumor is limited to the kidney and can be excised with the capsular surface intact. b) The tumor has undergone hematogenous metastasis, most commonly involving the lung. c) Prognosis is good with treatment. d) Prognosis is poor even with treatment.

• Prognosis is good with treatment. • The tumor is limited to the kidney and can be excised with the capsular surface intact. Long-term survival rates have increased to 90% for Wilms tumor stages I through III. The tumors usually are staged using the National Wilms' Tumor Study Group classification: stage I tumors are limited to the kidney and can be excised with the capsular surface intact.

A patient has developed acute tubular necrosis (ATN). The nurse knows that which of the following groups of drugs can cause this type of renal injury? Select all that apply. a) Chemotherapy drugs b) Radiocontrast dyes c) Aminoglycoside anti-infectives d) Nonsteroidal antiinflammatory drugs (NSAID) e) Angiotensin converting enzyme inhibitors (ACEI)

• Radiocontrast dyes • Chemotherapy drugs • Aminoglycoside anti-infectives Aminoglycosides, radiocontrast agents and chemotherapy drugs such as cisplatin (Platinol) all are directly toxic to the nephron and ATN will occur. -NSAIDs inhibit the synthesis of prostaglandins needed to maintain renal blood flow, thus renal perfusion declines and prerenal failure can develop. -ACEI are generally not nephrotoxic and are often used to manage hypertension in persons with renal disease.

A client is diagnosed with chronic kidney disease (CKD). The nurse knows that which of the following statements regarding CKD are correct? Select all that apply a) Onset is abrupt. b) Symptoms appear with 50% of nephrons lost. c) Renal damage is irreversible. d) Hypertension is a major cause. e) Less than 1% of population is affected.

• Renal damage is irreversible. • Hypertension is a major cause. CKD usually develops slowly, often over many years. Irreversible damage occurs, as symptoms do not appear until 80% of the nephrons cease to function. Approximately 10% of the United States population has CKD. Hypertension and diabetes are major causes of CKD.

A nurse is assessing a client diagnosed with CKD for neuromuscular manifestation. Select the manifestations the nurse may expect to find. Select all that apply. a) Loss of recent memory b) Restless leg syndrome c) Perceptual errors d) Increased muscle strength e) Increased alertness f) Peripheral neuropathy

• Restless leg syndrome • Loss of recent memory • Peripheral neuropathy • Perceptual errors • Increased muscle strength Many persons with CKD have alterations in peripheral and central nervous system function. Restless leg syndrome is a manifestation of peripheral nerve involvement, and muscle weakness and atrophy are a manifestation of uremia. Reductions in alertness and awareness are the earliest and most significant indications of uremic encephalopathy. These often are followed by an inability to fix attention, loss of recent memory, and perceptual errors in identifying persons and objects.

The nurse knows that a child with chronic kidney disease (CKD) may experience which of these manifestations? Select all that apply. a) Mental retardation b) Severe growth deficit c) High bone turnover d) Early sexual maturity e) Bladder incontinence

• Severe growth deficit • High bone turnover Childhood CKD is manifested by delays in growth and sexual maturity as a result of the uremic effects on endocrine function and bone growth. High bone turnover is related to secondary hyperparathyroidism. -Intelligence and control of urinary tract function are not directly affected by renal failure.

Acute tubular necrosis (ATN) is the most common cause of intrinsic renal failure. One of the causes of ATN is ischemia. What are the most common causes of ischemic ATN? (Select all that apply.)

• Severe hypovolemia • Burns • Overwhelming sepsis Ischemic ATN occurs most frequently in persons who have major surgery, severe hypovolemia, overwhelming sepsis, trauma, and burns. Hypervolemia and hypertension are not considered contributing factors to ischemic ATN.

The nurse recognizes that renal failure has many underlying causes, including which of the following? Select all that apply a) Renal disease b) Malnutrition c) Systemic disease d) Hepatic disease e) Nonrenal urological abnormalities

• Systemic disease • Renal disease • Nonrenal urological abnormalities Underlying causes of renal failure include renal or systemic disease and nonrenal urologic abnormalities. -Hepatic disease and malnutrition might gradually influence renal function as they can influence many body functions, but are not considered direct causes of renal failure.

The nurse will monitor clients with which of the following disorders for development of chronic kidney disease (CKD)? Select all that apply. a) Diabetes b) Polycycstic kidney disease c) Systemic lupus erythematosus d) Hyperlipidemia e) Glomerulonephritis

• Systemic lupus erythematosus • Polycycstic kidney disease • Glomerulonephritis • Diabetes Permanent renal damage can result from systemic lupus erythematosus, polycycstic kidney disease, glomerulonephritis, or diabetes. Hypertension also is a frequent cause for CKD. Hyperlipidemia does not cause CKD but may develop in clients with CKD.

A client is admitted with acute pyelonephritis. As the nurse enters the room, the client has shaking chills and reports acute pain. Other clinical manifestations the nurse will assess includes: Select all that apply. a) frequency, urgency, and dysuria. b) change in level of consciousness and hallucinations. c) shallow breathing and expiratory wheezes. d) moderate to high fever. e) pale skin and dry mucous membranes.

• moderate to high fever. • frequency, urgency, and dysuria. The onset of acute pyelonephritis is usually abrupt, with shaking chills, moderate to high fever, and a constant ache in the loin area of the back that is unilateral or bilateral. Lower urinary tract symptoms, including dysuria, frequency, and urgency, also are common. It is not associated with expiratory wheezes, hallucinations, or dry mucous membranes. (less)


Set pelajaran terkait

Geologic Time: Concept and Principles

View Set

Health, Chapter 16 "Tobacco", Section 3: Risks of Tobacco Use pp. 410-416

View Set

Unit 4 Quizzes (Ch. 24 The Urinary System)

View Set

Lymphatic and Immune Study Guide

View Set